Download as docx, pdf, or txt
Download as docx, pdf, or txt
You are on page 1of 73

2012

1. A 25 years old woman comes to emergency department, complaint with abdominal


pain and vomiting since 1 day. At the physical examination shows the patient looks
irritable, apatis, BP 80/50 mmHg, HR 130 x/m, RR 24 X/M, S 38.5 o C. At abdominal
examination, defance muscular (+), tenderness at all field of abdomen, bowel sound (-).
What is the condition in this patient?
a. Shock obstruction.
b. Shock cardiogenic.
c. Shock neurogenic.
d. Shock hypovolemic.
e. Shock anaphylactic.

2. As the diagnosis above, the first step you must do as the emergency doctor is:
a. Antibiotic infusion
b. Strong analgesic i.v
c. Abdominal plain X-Ray
d. Crystalloid rapid infusion
e. Adrenalin s.c

3. After doing that treatment, the hemodynamic of this patient is still unstable. The
Patient still have in shock condition. What is the next step you must do?
a. Change infusion fluid with Dextrose 5%.
b. Give vasoactive dopamine drug
c. Antipiretic drug i.v
d. Immediately Laparotomy
e. Antiemetic drug i.v

4. A 25 years old male presents to emergency with suspected closed left femur fracture
after motorcycle accident. He looks confused with blood pressure 80/60 mmHg and
pulse 110 /min. The doctor splints the fracture and gives large bores rapid crystalloid
and colloid infusion but unsuccessful in stabilizing the hemodynamic.
What is the most likely cause of the failure?
a. Inadequate splinting
b. Major artery injury of the left femur
c. Underlying heart disease  non cardiogenic
d. Ongoing bleeding in other parts of the body
e. No blood transfusion

5. A 35 years old female fall from 5 meters height. She presents with acute abdomen and
multiple open fractures on thigh and leg. Emergency laparotomy and temporary
external fixation is performed. After 6 days, the doctor wants to perform definitive
stabilization of the fracture. The reasons to wait until 6 days is to:
a. Achieve early healing of intraabdominal wound
1
b. Stabilize hemodynamic
c. Control infection
d. Avoid immunosuppression period
e. Reduce inflammatory reaction

6. A 45 years old male presents with multiple injury and unstable hemodynamic after
motorcycle accident. The doctor found that the lung is clear, no abdominal defans
muscular, stable pelvic and intact extremity
Where is the likely source of the bleeding?
a. Thorax
b. Intra peritoneal
c. Retroperitoneal
d. Intracranial
e. Pelvic cavity

7. A-17-year old woman comes to your private practice with pain on the left knee in
which the pain become worse at night for 2 months. Pain is aggravating by activity.
There was a history of injury at that knee. Sometime she feels fever and night sweat.
The physical examination shows a palpable mass, tender, warm. Range of motion of
the left knee is decrease. The radiographic finding shows a “sunray appearance” on the
proximal tibia. Which of the following is the most rational step?
a. Establish osteosarcoma as diagnosis
b. Give a broad spectrum antibiotic
c. Prepare for immediate surgery
d. Pain relieve after 2 weeks of treatment as an endpoint
e. Communicate with the patient, risk and benefit of treatment option

8. A-24-year old woman consulted to you that since 3 weeks ago she felt of constant
tiredness after delivery of her second child 15 months ago. She is slightly pale, but has
normal hemoglobin. Every day she works as a secretary from 8 am to 4 pm, her first
child now 3 years old. She has now returned because the tiredness persists and a friend
told her that a vitamin injection would do her good. That is what she wants. What
would be your treatment objective in this patient?
a. Advise her not to worry, it can resolve soon
b. Advise her to reduce physical/emotional overload
c. Doing a vitamin injection as she wants
d. Gives her the other option of drugs
e. Ask her to do other laboratory examination

9. A 37 year old man came to Emergency & Accident Unit with feeling of apprehension,
fear and palpitation, even mounting to feeling of impending death. After being
examined by doctor, the results showed that nothing was wrong. After 3 weeks, he got
the same feelings, and he had to go to the hospital again. But this time the Emergency
2
staffs were busy, and he was kept to wait. Until about 15-20 minutes he had not been
attended to, but his fear and palpitation subsided and became calm by it.
What would you think about the suffering that this person had?
a. Somatization disorder
b. Generalized Anxiety Disorder (GAD)
c. Phobic anxiety disorder
d. Panic disorder
e. Neurotic disorder

10. According to the above case, what would you do to help him?
a. Give some explanation as to the nature of his illness
b. Prescribe drug for this distress
c. Plan for psychoanalytical psychotherapy
d. Advise him to go for Yoga exercise
e. Advise him to go for refreshing

11. Again considering the above case, which of the following medication would suit him
well:
a. Diazepam
b. Chlorpromazine
c. Luminal
d. Alprazolam
e. Perphenazine

12. A 45 year old man is having the custom of drinking of beer. After some months, his
habit increase into heavier drinks such as wine and then even Whiskey and Vodka. Few
more months, he was admitted to a hospital because of an accident where he banged
his head when crossing a wooden bar where he fell unconscious for few minutes. He
was advised to be hospitalized. On the third day, he began to shake in his hands and
tremble the whole body. After being put on some drugs his shaking subsided and he
was discharged after being pronounced as recovered from his ailments. What was the
reason for his shaking?
a. Dementia
b. Brain contusion
c. Delirium tremens
d. Blackouts
e. Stroke

13. After his discharge from the hospital and returned to work he began to resume his
habitual drinking, his wife and family were against this habit of drinking, but he went
on disregarding the advice. Even he became irritable and shouted with abusive words
against anybody at home even he could beat his wife if she is nagging on. Some time he
would throw things and hit his children.
3
What would you call this episode of hitting family members?
a. Wife battering as a result of alcoholism
b. Alcohol intoxication
c. Marital discord
d. Personality disorder
e. Psychotic disorder

14. If in this family the children who used to be very dutiful at school works and compliant
to family discipline, observing father did a very bad example. The eldest son who is
rather big boy and physically robust became acting like a big boss against his
schoolmates who are smaller and weaker than him. Some time asking them to bring
him candies, some time he extorted money from them, otherwise he would threaten to
hit them or refusing to be his friends. What is the classification of this behavior
Personality disorder
a. A bully
b. Bossy
c. Delinquency
d. Genetic inheritance
e. ADHD

15. A 55 year old man came to emergency with suddenly being unconscious 2 hours before
admission after complaining of severe headache and vomit. His GCS was E3M5V3,
blood pressure was 190/100 mmHg, HR 90 x/min, RR 16 x/min, temperature 36,7 oC.
Which of the following examination is the most appropriate to exclude intracranial
bleeding?
a. Brain MRI
b. Angiography
c. Brain CT scan
d. Carotid Ultrasound
e. Electroencephalography

16. A 56 year old man was suddenly being confused and unable to move his left extremities
7 hours ago. On examination his blood pressure was 200/100 mmHg, HR 74 x/min
irregular. He was disoriented, left hemiplegia with increased tone on left side, and
babinsky sign on left side. Lab exam: WBC 12.000/mm3, RBG 190 g/dl. His CT scan
showed a large hypointense lesion in the right frontotemporoparietal lobe with
significant edema and midline shift. Which of the following is an appropriate
treatment?
a. r-TPA
b. Corticosteroids
c. Mannitol
d. Antihypertension
e. Antibiotic
4
17. For case above, if the ECG showed atrial fibrillation, which of the following examination
would be most appropriate for the next step?
a. Brain MRI
b. Brain PET scan
c. Echocardiography
d. Transcranial Doppler
e. Electroencephalography

18. A 17 year old boy is brought to emergency following fall from 4 meters height one hour
before admission. On examination he is alert, weakness of his left leg and loss of fine
touch on the left with loss of pain and temperature sensation on the right leg at level
Th-12 below. This clinical picture is most consistent with which of the following?
a. A posterior cord syndrome
b. An anterior cord syndrome
c. A complete cord syndrome
d. A central cord syndrome
e. A hemisection syndrome

19. Which of the following management is the most appropriate to prevent further
damage of spinal cord?
a. Give high dose corticosteroid
b. Trendelenberg position
c. Give mannitol
d. Neuroprotector agent
e. Lumbar corcet

20. A 52 year old man was sent to ER caused by seizure. His wife watched that he suddenly
stop his activity, starred, got involuntary movement on his right hand and then spread
to lower and upper right arm and finally became convulsion bilaterally. He has infarct
in left capsula interna with risk factor diabetes mellitus 3 months before and never
gets seizure. In the ER, the result of blood examination in normal limit, except blood
glucose was 58 mg/dl.
` Which of the following is most likely diagnosis at that time?
a. Symptomatic epilepsy
b. Pseudo seizures
c. Acute symptomatic seizures.
d. Idiopathic epilepsy
e. Secondary epilepsy

21. According to the case above, what is the type of his attack?
a. Partial seizure
b. Primary generalized seizure
c. Absence seizure
5
d. Simple partial secondarily generalized seizures
e. Complex partial secondarily generalized seizures

22. A 23 year old man was hospitalized after got head injury. His brain CT Scan showed
subdural hematoma and brain contusion in the right temporal region. On the
second days before underwent craniectomy, he suffered repeated left partial
seizures.
What is your working diagnosis?
a. Symptomatic epilepsy
b. Provoked seizures
c. Reflex seizures
d. Cryptogenic epilepsy
e. Pseudoseizure

23. A –PWE(patient with epilepsy)-42 year-old-man was treated with Phenobarbital 150
mg once daily but had bad compliance. During under psychological stress in his
office, he get repeated seizures. He brought to nearest hospital and during
transportation for 20 minutes he never gain his consciousness and still convulsive.
What is your working diagnosis?
a. Status epilepticus
b. Early phase status epilepticus
c. Acute symptomatic seizures
d. Panic attack
e. Psychological episodic attack

24. A 39 year old woman suffering from breast cancer since a year ago. She has main
complaint of headache that sometimes is accompanied with vomit for three weeks.
According to her husband, she gets tonic-clonic convulsion this morning. Which is the
priority diagnostic procedure you will perform?
a. Brain CT Scan with contrast
b. Brain MRI with contrast
c. EEG
d. Blood examination
e. Mammography

For Number 25-27


A 4 years old boy presents with burn wound on his stomach, hand, tight, and buttocks
after her mother accidentally spilled boiled water one hour before. Wound is felt pain, bullae(-).
He cried very loud, seems no dyspneu. Fever (-). History of immunization was unknown. The
patient never get this accident before. Alergy (-).
At physical examination, patient looks compos mentis, moderate ill. Vital sign is normal.
Weight is 12 kg. At thorax examination, chest retraction (-), syanosis at extremity (-), and good

6
perifer perfusion. Localis status of this patient show pink on the wound base, tepi tidak rata,
necrotic tissue (+), bullae (-), pus (-), brown/black patchy (-), abses (-). dengan luas 21%.
Laboratory examination shows anemia and mild leucocytosis.

25. What is the degree of that burn trauma on this patient?


a. 1
b. 2
c. 3a
d. 3b
e. 3c

26. What is the best thing you must do for this patient before bring him to the hospital?
a. Give tooth pasta
b. Close with sterile dressing
c. Shower with water for 15 minutes
d. Compress with ice for 15 minutes
e. Give topical antibiotic

27. What is the initial step you must give at hospital?


a. Fluid Infusion Maintenance
b. Give topical antiseptic
c. Antibiotic
d. Parlkland resuscitation infusion
e. Bulectomy

For Number 28-29


A 34-year-old woman just delivered a 4100 g boy after a 15-hour labor, including a 2,5 hour
second stage. During the repair of a midline episiotomy, there is a marked increase in the
amount of vaginal bleeding.
28. Which of the following is the most common cause of immediate postpartum
hemorrhage?
a. Retained placental fragments
b. Uterine atony
c. Cervical laceration
d. Vaginal laceration
e. Disseminated intravascular coagulation

29. Which of the following is the best immediate management of the probable cause of
this postpartum hemorrhage?
a. Massage and compression of the uterine fundus
b. Intravenous administration of 20 units of oxytocin
c. Abdominal hysterectomy
d. Uterine artery embolization
7
e. Hypogastric artery ligation

30. Dedy, 13 year old man brought to Emergency by his mother with seizures and altered
consciousness. His mother said that his seizures happen continuously since 45 minutes
ago. Two days ago, he also suffers seizure about 40 minute. Physical examination:
Temp 39.5⁰C, He was lethargic, BP 120/80 mm Hg, Heart Rate 110 X/minute. Pupils
were reactive and no obvious focal deficits. Babinsky sign present bilaterally. Lab
examination: Hb 14 mg/dL, Leukocyte 11.000 /mm3, Random glucose: 70 mg/dL, Liver
Function Test, Renal function, Electrolyte: in normal limit.
According to your examination which of the following is the most likely diagnosis?
a. Febrile seizure
b. Status epilepticus
c. Cryptogenic seizure
d. Refractory epilepsy
e. Hypoglycemic seizure

31. A 40 years old man presented in Emergency unit (ER) with unconsciousness. His wife
finds him in the bath room 2 hours ago and brings him to the hospital. Physical
examination: Unconsciousness, no trauma, BP 160/95 mm Hg. What kind of laboratory
examination is the most appropriate to find non structural causes of unconsciousness?
a. Na , K , Ureum
b. Na, K, Glucose
c. Na, Glucose, Ureum
d. Na, Cl , Glucose
e. K, Cl, Glucose

32. A 7 years old boy, have injury falls from bicycle and had wound and bleeding at the
lower extremity. What is the most appropriate sequent to stop bleeding :
a. Vascular spasm – Blood coagulation – Platelet plug formation
b. Vascular spasm - Platelet plug formation - Blood coagulation
c. Platelet plug formation - Blood coagulation – Vascular spasm
d. Blood coagulation - Vascular spasm - Platelet plug formation
e. Blood coagulation – Platelet plug formation – Vascular spasm

33. A 60 year old man came to emergency unit with history of chest pain 9 days ago. Which
is the most appropriate laboratory examination do you need for this patient?
a. CPK serum
b. CK MB serum
c. Troponin I and T
d. SGOT/AST
e. Myoglobin

8
34. A 31 year old man came to emergency depart with complaint severe pain in abdominal
area and frequent vomit since 24 hours ago. In physical examination looks severe pain,
pain pressure abdomen (+) in all area. Radio imaging: not find sign of perforation.
What’s the most important laboratory examination do you need for make diagnosis?
a. Urinalysis
b. Amylase serum
c. Lipase serum
d. Widal serology
e. Liver function test

35. A 22-year-old female college student is brought into the emergency


room by the police, who found her walking back and forth across a busy
street, talking to herself. The young woman appears to be oriented
with respect to person, place, and time. Her first hospital admission
was two months ago for a similar condition. During a psychiatric
interview, she has difficulty concentrating, and seems to hear voices
A phone call to her sister provides the additional information that
the girl dropped out of school three months ago and has been living on the street. Urine
toxicology is negative. What is the most likely disorder?
a. Schizoaffective
b. Halusination
c. Bipolar delusion
d. Schizophreniform
e. Panic

36. A 45 year old man fell down after complaining severe headache. Vital sign was normal.
He was vomits several times after and had meningeal signs. What is the most
appropriate investigation needed?
a. Transcranial doppler
b. Laboratory
c. CT scan
d. Cranial X ray
e. Electroencephalography

37. A 20-year female presents with abdominal pain which was initially felt in the centre of
abdomen but now felt in the right iliac fossa. She denies any other symptom. Her last
menstrual period was 5 days ago. Examination reveals low fever (37.9°C) and tender
right iliac fossa. Digital rectal examination shows tenderness on 9-12 Clock direction.
What is the most likely diagnosis?
a. Acute appendicitis.
b. Mid-cycle pain (Mittelschmerz).
c. Urinary tract infection.
d. Mesenteric lymphadenitis.
9
e. Intestinal obstruction

38. An unconscious patient with sepsis in the intensive care unit undergoes a 2-hour period
of severe hypotension. Blood chemistries taken during the following 48 hours show
rising creatine kinase MB fraction (CK-MB), peaking at 5 times the upper limit of
normal. ECG findings are equivocal; with some degree of flat S-T segment
depression over several leads. Which of the following is the most likely diagnosis?
a. Prinzmetal angina
b. Stable angina
c. Subendochondral infarction
d. Transmural infarction
e. Unstable angina

39. A 74-year-old woman, in otherwise good health, tripped and injured her right leg 2
days previously and has been bedridden since the accident. Two hours ago, she
suddenly had shortness of breath and became delirious. On physical examination, her
blood pressure is 120/70 mm Hg, heart rate is 110, and respiratory rate is 32 x/min.
Pulse oximetry shows an oxygen saturation of 80%.
Which of the following is the most likely diagnosis?
a. Pneumonia
b. Acute cerebral infarction
c. Myocardial infarction
d. Pulmonary infarction
e. Pulmonary thromboembolism

40. A 60 year old woman comes to emergency room, complaining about dyspnea,
dizziness, fatigue, lightheadedness and palpitation. After a few second, the patient
collapse. ECG shows on Lead II

What is the initial management for this patient?


a. Nitroglycerin sublingual
b. Echocardiograph
c. Heart surgery
d. Dopamin
e. Defibrilator

10
41. A 61 years old man comes with vomitting, dispepsia and sweating. He feels cool when
he plays tennis and fainting for a few minute. He has hypertension and diabetes, no
smoking and no hypercholesterolemia.

What is the most likely location of AMI?


a. Inferior
b. Posterior
c. Anterior
d. Lateral
e. Anteroseptal

For Number 42-45


A 60 year-old man, is brought by his friend to ED because of unconsciousness. According to his
friend, the patient is a heavy smoker and often to be hospitalized due to shortness of breath.
On physical examination, RR: 42x/m, BP: 100/80, Heart rate 120x/m. Blood gas analysis
showed pH: 7,20 ( 7,35-7,45); pO2: 50 (>80); pCO2: 82,5 (35-45); HCO3: 28 (22-26); BE +3 ( 2);
SpO2: 88% (>95). Routine blood examination: Hb: 17.8; Ht: 51; WBC: 11.000; Platelet 420.000,
Natrium: 130 (135-145); Kalium 5 (3.5-4.5). The chest X-ray showed lung hyperinflation.

42. According to those data, what is the most possible cause of the unconsciousness?
a. Severe hypoxemia
b. CO2 narcosis
c. Sepsis
d. Hypokalemia
e. Hyponatremia
43. What is the lung function test might be showed?
a. FEV1/FVC < 70
b. FEV1/FVC > 70
c. FEV1/FVC normal
d. FEV1 > 75
e. FVC > 80

11
44. If we measure the lung static volume, what is the most possibility result?
a. increased tidal volume
b. increased expiratory respiratory volume
c. increased residual volume
d. decreased functional residual capacity
e. decreased total lung capacity

45. What is the most inflammation cell that plays a role?


a. Macrophage
b. Eosinophil
c. Neutrophil
d. Mast cell
e. Basophil
Airway tissue neutrophils are increased in the large and small airways during infection
and exacerbations, whilst parenchymal neutrophil numbers are inversely related to
alveolar wall destruction, suggesting that they are not involved in the progression of
emphysema. Macrophages are increased throughout the respiratory tract airway lumen
and epithelium in COPD and are positively related to severity of disease, airway
obstruction and degree of alveolar wall damage in emphysema

For Number 46-47


46. A 32 year old man brought by his wife to mental hospital. The wife complained that
since 3 days ago, her husband had disorganized behaviour and speech. In this case, the
doctor gives Haloperidol 3 x 5 mg a day. After 2 days, his wife come again and reported
that he showed shaking his hand and feet.
What does he suffered from?
a. Acute Dystonia Reaction
b. Akathisia
c. Torticolis
d. Parkinsonism
e. Tardive Diskinesia

47. According to the case above, what medicine should be given to the patient to reduce
the symptoms?
a. Metilphenidate
b. Triheksiphenidil
c. Risperidone
d. Chlorpromazine
e. Carbamazepine

For Number 48-50


A 17 year old girl attempted to suicide by drinking one cup of “Baygon”. Her mother found her
diary saying that she really wants to be dead because her boyfriend has another girl and leaved
her.
12
48. What is the most probably diagnosis?
a. Anxiety Disorder
b. Panic Disorder
c. PTSD
d. Schizophrenic
e. Depression
49. According to the cases above, which type of the suicide she has?
a. Suicide Attempt
b. Suicide Gesture
c. Suicide Gamble
d. Suicide Equivalent
e. Suicide Attack

50. What is the classification of suicide for the case above?


a. Euthanasia
b. Murder Suicide
c. Self Injury
d. Attack Suicide
e. Mass Suicide

13
2016
30 yo woman experiences impulsive wishes pengen slash her wrist, generalized anxiety, social phobia,
dissociation. Exam: no psychotic symptoms. Waktu Kecil ada sexual dan physical abuse. Diagnosis?
a. Depresi
b. Schizophrenia
c. Generalised anxiety disorder
d. Ptsd
e. Ocd

Anka 6 th demam 39C ada seizure selama 10-15 menit. Sekarang seizure 20 menitan tp blm berhenti.
Ortu kasi diazepam rectal. Terus dibawa ke RS. Di RS dikasih diazepam IV tp ga membaik. Penanganan
selanjutnya?
a. Kasih diazepam lagi
b. 5mg/ kgBB fenobarbital
c. 20mg/kgBB fenitoin
d. EEG
e. LP

Pasien ke tabrak 65kmph terus ada fraktur di tibia, pain pelvis, dyspnea. Dia ada tension pneumothorax
terus ada di needle (Breathing). Udah dikasih warm crystaloid buat circulation, next mau diapain?
a. Urethro and pyelogram
b. External pelvic fixation
c. CT abdomen
d. Foto apa gitu

Primary survey: A,B,C,D,E + adjuncts

Berdasarkan kasus diatas (status epilepticus), dikasi obat apa?


a. Anticonvulsant drug
b. antiseizure drug
c. antiepileptic drug
d. drug

Anak tiba” kejang, sebelumnya ada riwayat kejang 2x sejak 2 bulan lalu. Diagnosis? Epilepsi

Lanjutan nomor diatasnya (anaknya kejang 2 menit dan berhenti dibawa ke RS kalo ga salah udh dikasi
diazepam rectal 1x)
a. Anaknya udh 3 kali kejang tnp conscious, mau dikasi a[a?
b. Diazepam IV krn pasien baru dikasih diazepam rectal
c. Phenytoin IV

Ahmad, 65 th, 1 th sblnya anterior myocard infract. Keluhan SOB kalo. Jalan jauh + naik tangga. Tp
aktivitas seharinya tetap OK. NYHA class berapa?
a. NYHA 1
b. NYHA 2
c. NYHA 3

14
d. NYHA 4
Orang melena, diffuse pain abdomen awalnya cuma di epigastric, riwayat gastritis kronik sama artritis +
pake pain killer, terus dia hypovolemia.
pasien gastritis karena pake pain killer NSAIDs
Kenapa? Perforasi gaster karena pemakaian NSAID (pakai NSAIDs krn dia artritis)
Initial? Resusitasi cairan . 1 L Warm Crystalloid , untuk cairan maintenance
gunakan holiday segar ( 4 2 1)
Definitif? Laparotomi eksplorasi

Management of the above (no 24) - pokoknya kasus peritonitis et causa appendicitis. Jawaban: IV fluid,
antibiotics analgesic

Lanjutan no 2. Di tnya mechanism? Diffuse peritonitis karna app

Male kena tembak single bullet wound, managementnya apa? Transfer to operating room while giving
fluid

Ada edema ekstremitas bawah. Minum furosemide. Lalu mengalami syncope. Type syncope? drug
induced

Orang cari attention buat suicide= suicide gesture


Treatment= psychoterapy
Pyschopathic disorder

Ada orang G3P3A0 melahirkan anak 4200 gram, dalam waktu 15 menit keluar, kala 3 placenta keluar
dalam 5 menit. Terus hemmorhage 1 jam setelahnya. Masalah PPH apa?
Atony bisa jadi karena multipara, tapi biasanya ada retensi plasenta
Vaginal laceration anaknya besar / macrosomia
Coagulopathy

Orang dtg 3a.m ke A/E department singing loudly and throwing kisses. Katanya dia dari jam 10 singing
and shouting loudly. Kata istrinya dia stop minum obat (ga disebut obat apa). 2 weeks ago katanya full
energy. Kenapa ini org?
a. Heroin intox
b. Schizophrenia
c. Alcohol intox
d. Acute mania
e. Cyclothymia

3 kali seizure Tanpa regain of conciousness di antaranya. Ini apa?


a. Epilepsy
b. Status epilepticus karena udah lebih dari 2x dan dia ga regain conciousness, atau dia terus
terusan dalam waktu 5 menit.
c. Satutus convulsius
d. Seizure
e. Convulsion

15
Male 19 th, present with seizure. He regain conciousness GCS 15, 3/5 right extremities muscle weakness.
What is the next step?
a. CT scan head bisa untuk liat kecurigaan ada lesi di otak
b. EEG untuk pastiin seizurenya beneran apa boongan

Burn patient 2nd degree burn on left hand, lower arm, and upper arm, what is the burn percent? (Rule
of nine) 4.5 % kalo cuma salah satu sisi (anterior / posterior) , 9% kalo udah kena anterior dan
posterior

Acetaminophen bisa bikin kerusakan hepatocyte (gejala: jaundice). Antidote acetaminophen


a. N -acetylcysteine
b. Metylprednisolone

Symptoms: ipsilateral paralysis/ motor loss, ipsilateral propioception (position sense) loss, and
contralateral pain and temperature loss.
a. Hemisection syndrome
b. Anterior cord syndrome
c. Posterior cord syndrome
d. Central cord syndrome
e. Complete cord syndrome

Pasien perempuan 14 th, didiagnosis tonic clonic seizure tapi kurang toleran (obat apa?) dengan obat.
Dia dibawa ke a/e dgn status epilepticus, apa obat yang tepat buat dia?
a. IV diazepam
b. IV phenytoin
c. IV phenobarbitone
d. IV tiopental

Pokoknya ada ibu datang keluar poop dari vagina karena ke dukun
a. Perineal tear grade 1
b. Perineal tear grade 2
c. Perineal tear grade 3
d. Perineal tear grade 4
e. Perineal tear grade 5

36 thn, came to company clinic after work accident, splashed her leg and foot with falling chemical
container. She is in great pain and crying for help. First step?
a. Give pain reliever
b. Commence water irrigation
c. Contact plastic surgeon on call
d. Test chemical with litmus paper

Cowo tiba-tiba chest pain saat nonton TV, pain last for 30 mins, dibawa ke RS hilang. Punya history of
abnormal RBG (diabetes bisa menyebabkan ACS) , diagnosis?
a. ACS
b. Mitral stenosis
16
c. Aortic regurgitation
d. Acute pericarditis
e. Lupa

Pasien laki-laki 34 thn, kecelakaan motor, ga pake helm, jatuh kena kepala di jalanan. Tidak sadarkan
diri. BP: 120/80 HR:88. Kenapa ga sadar?
a. Retak tulang kepala krn ga pake helm
b. Hematoma compresi brainstem bisa menekan ARAS
c. Contusion
d. Lupa
e. Heart attack, other medical condition

Maksimal dosis misoprostol? ( 1 mg = 1000 microgram)


c. 1200 microgram 800 - 1000 microgram kalo di FIGO
d. 1200 mg

Pasien dengan nyeri dada kiri pertama kali, sebelumnya ga pernah kek gini (ga ada keterangan di soalnya
berapa menit membaik dan membaik atau makin parah kalo lg ngapain)
a. Unstable angina hrsnya CCS grade 3
b. Stable angina
c. MI harus ada data enzim jantungnya

Pasien dtg dgn gejala intoksikasi opioid pada pemeriksaan tampak nafas pasien dangkal. Manajemen
utama?
a. Secure airway
b. Administer naloxone

What is the mechanism of migratory pain in acute appendicitis?


a. The visceral peritoneum is inflamed first so it causes pain in epigastric area

Infection and source control pada peritonitis diffuse ec app perforasi? Exploratory laparotomy

Itung GCS
Dekortikasi fleksi abnormal
Deserebrasi ekstensi abnormal

Anak kejang demam 3 kali dalam 1 tahun. perlu dikasi obat apa?
b. Paracetamol
c. Valproic acid
d. Tidak perlu diberi obat antikejang

Boxer kena tinju KO di kepala, pertama unconcious, lalu sadar dan perlahan jadi tidak sadar (lucid
interval). Kemungkinan diagnosis? EDH

Korban trauma , GCS 6. Apa tindakan pertama? Secure airway: ETT (GCS < 8)
17
Pasien datang dengan kejang generalised. 2 tahun sebelumnya ada riwayat stroke. Diagnosis?
a. Epilepsi
b. Acute symptomatic seizure

SOB and wheezing, next management?


a. Cek airway
b. Curiga pasien asma

Child SOB, can’t take food and fluid regularly, and is drooling (kemungkinan epiglotitis). Management?
….
Airway: oksigen, secure airway with ETT, lihat epiglotitis pake imaging utk pastiin diagnosis (thumbprint
sign)
ETT: inability to swallow, drooling, respiratory distress, stridor
bisa sakit antara epiglotitis atau difteri (tp difteri biasanya pasien udh parah, udh mau mati)

18
2017
1. A 53 years old patient complained chest pain admitted to Emergency room. After performed a
series of physical examination, an ECG test revealed an anterior myocardial infarction. What artery
is the most responsible for that condition?
a. Marginal Coronary Artery
b. Circumflexa artery
c. Posterior Descending interventricular artery
d. Right Coronary Artery
e. Left anterior descending artery -> myocardium anterior dapat supply dari left anterior
descending coronary artery

2. A female 29 year old patient came to the Emergency Department with generalized tonic clonic
seizure that lasted for 2 minutes. The seizure continued for 3 times with no regain of consciousness
between seizures. What would be your drug of choice treatment for the diagnosis above?
a. Phenobarbital intravenously
b. Phenytoin intravenously
c. Diazepam intravenously -> treatment of choicenya lorazepam (benzodiazepines) 0.1mg/kg IV
maks 2mg/min, kalau tidak tersedia bisa diganti diazepam 0.15mg/kg IV maks 5mg/min terus
bisa repeat dose 3-5 menit kemudian jika kejang belum membaik | rute utamanya prefer IV
d. Propofol intravenously
e. Carbamazepine intravenously

3. A male 65 year old patient came to the Emergency Department with seizures starting from the right
extremities then continue to all four extremities, his eyes and head turned to the right side and he
was non responsive during his seizure, which lasted for 3 minutes. After that, he regained full
consciousness. He had a history of stroke with right hemiparesis since 2 years ago. EEG revealed
abnormality over the left temporal region. What would be the most best drug of choice treatment
for the diagnosis above?
a. Anti convulsant drug orally
b. Anti epileptic drug orally -> tetap prefer ini untuk management epilepsy soalnya pasien stroke
yang ada chronic/recurring seizure dianggap epilepsy | first linenya
carbamazepine/lamotrigine/sodium valproate/topiramate
c. Anti stroke drug orally
d. Anti status epileptic drug orally
e. Anti seizure drug orally

4. A patient has a blood pressure of 70/50 mmHg and a serum lactate level of 30 mg/100 mL (normal
6-16). His cardiac output is 1,9 L/min, and his central venous pressure is 2 cm H2O. The most likely
diagnosis is:
a. Congestive heart failure
b. Septic shock -> CO normal/elevasi
c. Distributive shock/septic shock

19
d. Hypovolemic shock -> CVP bisa menurun pada hypovolemia/venodilation (>10% darah) jadi
mungkin blood lossnya udah tinggi + cardiac output turun di hypovolemic; cardiogenic;
obstructive
e. Cardiac tamponade

5. An electrician is electrocuted by a downed power line after a thunderstorm. He apparently made


contact with the wire at the level of the right mid thigh. In the emergency department, his vital
signs are normal and no dysrhythmia is noted on ECG. On examination, there is an exit wound on
the bottom of the right foot. His urine is positive for blood by dip stick but no RBCs are seen
microscopically. Initial management should include:
a. Immediate angiography
b. Aggressive fluid infusion
c. Debridement of necrotic muscle
d. Admission to the intensive care unit for observation
e. Intravenouspyleography

6. Recently, We heard news from television that there was a bomb exploded at a big mall in the city.
As the consequence, many people was died there at that moment overall. Which of the answer is
the most correct one in this case?
a. Assisted suicide
b. Attempt suicide
c. Mass suicide -> biasanya ada paham tertentu
d. Murder suicide
e. Suicide attack -> bunuh orang lain + dia

7. Suppose in the emergency room, his skull X ray shows linier fracture at the left temporal region,
and the physician decided to do observation for 6 hours, but after 2 hours he became unconscious
again. Now his BP increase to 150/95 mmHg, pulse up to 100 x/minutes, respiratory rate up to 20
x/minutes. The pupil on the left 4 mm very slow response to the light, and right pupil 2 mm
response to light. His left arm and leg are inactive compare to the right side. How do you explain his
left arm and leg became in active compare to the right side?
a. The right midbrain is compressed either by blood clot or the swelling brain
b. Right brain are pushed and shift to the left and the left midbrain are compressed the tentorial
edge
c. The left brain are pushed and shift to the right and the right midbrain are compressed the
tentorial edge -> pupil ipsilateral dilatasi + hemiparesis bisa contralateral/ipsilateral =
uncal/transtentorial herniation
d. The left midbrain is compressed either by blood clot or the swelling brain
e. The hematoma cause compression on both side of the brainstem

8. A 29-year-old woman presents with massive post-partum bleeding. She is gravida 3 para 3 and has
just delivered a 4200g (9 lb 4 oz) infant vaginally after a 5 hour labor. The delivery itself was

20
spontaneous after 15 minutes of pushing. The placenta was delivered 5 minutes later with minimal
traction. What is the most likely cause of her bleeding?
a. Coagulation defect
b. Uterine atony -> persalinannya terlalu cepat
c. Chorioamnionitis
d. Vaginal lacerations
e. Retained products of conception

9. A 23-year-old man is brought immediately to the emergency department from the hospital' s
parking lot where he was shot in the lower abdomen. Examination reveals a single bullet wound. He
is breathing and has a thready pulse. However, he is unconscious and has no detectable blood
pressure. Optimal immediate management is to:
a. Initiate infusion of packed red blood cells.
b. Initiate primary survey and transfer the patient to the operating room
c. Perform diagnostic peritoneal lavage.
d. Initiate fluid therapy to return his blood pressure to normotensive
e. Insert a nasogastric tube and urinary catheter.

10. A 6-year-old boy developed seizure and fever to 39 ° C. He had multiple generalized seizures,
ranging from 10 to 15 minutes. The last seizure had occured for more than 20 minutes and had not
stopped. His parents had given 2 doses of rectal diazepam at home. The boy was brought to
emergency department. An intravenous diazepam was given but the seizure persisted. What is the
most appropriate management for this patient?
a. Give 20 mg/kg intravenous phenytoin
b. Give 5 mg/kg intravenous phenobarbital
c. Do an electroencephalography
d. Do a lumbar puncture
e. Give another dose of intravenous diazepam

11. A 30-year-old woman is brought into your clinic with burns affecting both of her entire upper
extremities and the anterior surfaces of both legs after spilling boiling soup from the stove while
cooking approximately two hour ago. Her weight is 48 kg, height 150 cm, blood pressure 90/60
mmHg, heart rate 100 bpm, respiratory rate 20 x/min. Using Wallace's Rule of Nines what is the
percentage area burnt?
a. 27%
b. 36% -> whole upper extremities (4.5 + 4.5 + 4.5 + 4.5) & anterior surface both legs (9 + 9)
c. 18%
d. 45%
e. 54%

12. A 34 year old female came to your Emergency department with profuse bleeding. She has just
deliver a twin baby half an hour ago weight 2800 grams and 2600 grams, this is her 6th pregnancy.
The patient is unconscious, BP 80/palp, HR 110 beats /min, when you inspect the vagina, heavy
21
bleeding still occurs. You decided to administered misoprostol to stop the profuse bleeding. What is
the maximum dose of misoprostol that you can admit in 24 hours?
a. 1200 μg -> microgram
b. 800 mg
c. 600 mg
d. 1200 mg
e. 800 μg

13. A 30-year-old woman experiences impulsive wishes to slash her wrists, generalized anxiety, social
phobia and dissociation. Although examination reveals no psychotic symptoms, she reports having
recurring flashbacks of childhood sexual and physical abuse. What is the most likely diagnosis?
a. Generalized anxiety disorder
b. Depression
c. Obsessive-compulsive disorder
d. Schizophrenia
e. Post-traumatic stress disorder

14. A 37 year old G5P3A1 came to the emergency department because of vaginal bleeding, and
abdominal cramps. History is unrevealing except for an induced abortion. Physical examination
reveals a BP of 110/70 mmHg, pulse 120 x/mnt, and temperature 38.50C. The abdomen is tender
with slight rebound in the lower quadrants. The pelvic examination reveals blood in the vault and a
foul smelling discharge from the cervix, which is dilated to 1 cm. The uterus is 8 weeks size and
tender. What do you think happen in this patient and what is the treatment?
a. Septic abortion and curettage -> dischargenya bau
b. Ectopic pregnancy and laparotomy
c. Uterine perforation and laparotomy
d. Molar pregnancy and laparotomy -> abnormal vaginal bleeding + uterus lebih besar dari
ukuran normal + nyeri dari large benign theca-lutein cysts + vaginal passage yang menyerupai
vesikel anggur +
e. Missed abortion and curettage

15. A 23-year-old female receptionist presents to her general practitioner one hour after having
collapsed at work. She was standing at a counter when she suddenly felt lightheaded and
remembers having distorted hearing. She then lost consciousness and fell to the ground. Her
colleagues reported seeing her limbs stiffen briefly. After 10 seconds she regained consciousness
and she recalls seeing her colleagues standing over her. Which of the following is the most likely
diagnosis?
a. Multiple sclerosis
b. Cerebral tumor
c. Complex partial seizure/focal onset impaired awareness/focal impaired awareness
d. Narcolepsy -> gangguan sleep-wake pattern jadi pasiennya ada excessive daytime sleepiness
e. Vasovagal syncope -> gangguan sympathetic outflow jadi timbul penurunan sympathethic +
increase parasympathetic

22
16. 34 year old G5P2A2 came to your clinic due to abdominal pain. It’s worsening since tonight and
radiating to her left shoulder. She feels the pain about 2 weeks on and off. She denied any delay of
period and has regular menstruation although it’s only spotting since she is using oral contraception
pills for breastfeeding. She has delivered her last baby 1 year ago spontaneusly. She denied vaginal
bleeding. Her vital signs: BP-110/90; HR-120x/mnt; RR-24x/mnt; T-36.50C. There is muscular
defense and tenderness all over the abdomen. On internal examination is firm with cervical motion
tenderness. A qualitative b-hcg was done and the result was positive. What do you think happen in
this patient?
a. Molar pregnancy
b. Ectopic pregnancy -> dia ada pain + penggunaan kontrasepsi
c. Inevitable abortion -> ada vaginal bleeding + cervical os sudah kebuka sebagai tanda
conception product bakalan keluar dalam waktu dekat
d. Imminens abortion
e. Complete abortion -> ada vaginal bleeding + produk konsepsi yang sudah keluar via cervix

17. A 24-year-old boxer was brought to emergency room with unconsciousness. Based on paramedic’s
story he was knocked out during a match after getting hit on his temple. He was initially
unconscious but regains his consciousness several minutes later. On the way to the hospital there
was traffic jam, and he appears to be drowsier until he gives no response. On examination you
found under pain stimuli his eyes are not reacting, he produce incomprehensible sounds,
withdrawal reflex from the stimuli and unequal pupil. What is your first step to help this patient?
a. Obtain emergency head CT-scan
b. Give him oxygen 10 L/min via non-rebreathing mask
c. Begin CPR sequences
d. Secure definitive airway
e. Refer to a neurosurgeon

18. A 34 year old female came to your Emergency department with profuse bleeding. She has just
deliver a twin baby half an hour ago weight 2800 grams and 2600 grams, this is her 6th pregnancy.
The patient is unconscious, BP 80/palp, HR 110 beats /min, when you inspect the vagina, heavy
bleeding still occurs. What’s the most common possible cause of the bleeding?
a. Uterine rupture
b. Perineal tear
c. Uterine atony -> kelahiran kembar = tone
d. Retained placenta
e. Blood coagulation disorders

19. A 35-year-old, G4 P3 A0, woman has just delivered a live male infant weighing 4.5 kg vaginally after
induction of labour for gestational hypertension. Following delivery of the placenta, a rapid loss of
1000 mL of blood occurred. Immediate bimanual examination of the uterus reveals it to be poorly
contracted. Which of the following drugs is best for controlling haemorrhage in this woman?
a. Oxytocin
b. Misoprostol
c. Vasopressin
23
d. Methylergometrine -> bisa dikasih pada pasien pasca persalinan placenta tapi kontraindikasi
buat hipertensi
e. Syntometrine

20. A 26-year-old woman present to Emergency Department after giving birth to a healthy 2800 gr
baby with help from Traditional birth attendance/dukun yesterday. Her family says that after the
baby was delivered, it took nearly 1 hour until the placenta is delivered. She is drowsy and less
responsive, BP 60/40 mmHg, HR 136, RR 28 x/min, with cool and clammy skin. She is brought
because there is continuous trickle of blood from her vaginal. Which of the following is the next
step in managing this patient?
a. Vitamin K injection
b. Antibiotics
c. Analgesia
d. Elective evacuation of the uterus -> kating
e. Volume resuscitation -> dia udah hipotensi + pulse rising jadi resusitasi dulu

21. A 78 years old female is admitted to ER because she has difficulty in breathing. She is a diabetic
mellitus type 2 patient, she took her diabetic medicine on and off. She has history of difficulty of
breathing since 1 week ago, starting with mild cough with thick cloudy sputum and continuous
fever, treated with antibiotics from local doctor. She is now alert, agitated breathing heavily with
could speak one or two words only, with BP 110/60 mmHg, HR 110x/m, regular and equal pulse and
warm extremities. RR 36x/m, Temp 38.7 C, O2 Sat 92 % with room temperature. What prioritized
step should you do next?
a. Give her intravenous broad spectrum antibiotics to help reduce the infection
b. Give her intravenous antipyretics to reduce the fever
c. Give her high level oxygenation to help her breathing
d. Give her sedation, because she is agitated
e. Give her intravenous bronchodilator and mucolytic to help reduce the sputum to get out

22. A 70 y.o. female is admitted to ER at 11 PM with chief complain diffuse/whole severe abdominal. At
first the pain was only in her epigastric area for 4 days and then it spread to whole abdomen for the
last 2 days. She also have continuous progressive fever since this morning, with very minimal oral
intake since this morning also. Since 3 days ago she has melena, without diarrhea, with normal
stool volume and frequency. She also has decreased micturition. No history of abdominal trauma,
no history of previous episode like this. She has history of chronic gastritis and chronic knee
arthritis. She routinely consumed painkiller for her knee arthritis, bought by the patient herself
(self-medication) for more than 10 years. She now looks sleepy and lethargic with GCS E3M6V5 BP
100/60 mmHg, HR 120x/m, RR 20x/m, Temp 38C. Skin is dry with decreased turgor. Extremity is
cool, clammy with normal capillary refill. Abdomen is distended, negative liver dullness, decreased
bowel sound, muscular rigidity is present with tenderness and rebound tenderness at palpation.
Digital Rectal Examination/DRE shows that she has decreased sphincter tone, smooth mucosa,
rectal vault is normal with tenderness all direction. The punctum maximum of pain is at the
epigastric region. At your gloves after DRE you found black bloody stool with foully smelling. What
do you think happened with the patient?
24
a. Hypovolemic shock due to bleeding from the gastric ulcer
b. Acute appendicitis with perforation, causing diffuse peritonitis
c. Diffuse peritonitis due to perforated peptic ulcer, now with sepsis, possibly severe dehydration
d. Acute bowel obstruction due to colonic tumor
e. Diffuse peritonitis due to perforated colonic tumor

23. A 56 year-old male with longstanding uncontrolled diabetes was admitted to emergency unit with
decreased consciousness since 2 days ago. He also had wound on the sole of his right foot since 2
weeks ago. On physical examination he was somnolen, with BP 80 mmHg/palpation, pulse 120
bpm, respiratory rate 34 times/minute, temperature 38°C. His extremities were cold and clammy.
There was an ulcer in the sole of his right foot, diameter of 5 cm, dirty with pus and necrotic tissue.
The examination to determine the etiological cause of the infection is:
a. Blood leucocyte with differential count
b. Procalcitonin
c. Wound culture
d. Blood culture
e. C-reactive protein

24. A 40-year-old woman presented with subacute onset of quadriparesis of 3 months' duration,
associated with electric shock like pains radiating along the outer aspect of the right arm. On
examination, she had spastic grade 4/5 quadriparesis, the right biceps jerk was absent, and the
other tendon jerks were brisk with extensor planter responses. What is the most likely diagnosis?
a. Guillain-Barre syndrome -> tapi ga ada riwayat pemicu
b. Subacute combined degeneration of the cord -> dominan proprioceptive & vibration loss
c. Nerve root compression at C3 level
d. Motor neuron disease -> bizzare distribution
e. Cervical cord compression at C5 level -> electric shock sensation

25. A 56 years old female came to ER with wound on the right foot sole since a week before. She felt
nausea and did not eat since 3 days ago. She only drink enteral diet for diabetes. She had diabetes
since 5 years ago and was treated with metformin 500 mg t.i.d., and she also had history chronic
heart failure treated with aspirin 80 mg once daily and furosemide 40 mg once daily. On physical
examination, her blood pressure was 80 mmHg/palpation, heart rate 130 beats perminute,
respiratory rate 40 times perminute and temperature 39°C. Her extremities were clammy and cold.
Laboratory revealed hemoglobin level 7.5 g/dL, leucocyte 26,000/μL, blood glucose 349 mg/dL,
blood ketone 0.9 g/dL. What is the mechanism of current main problem in this patient?
a. Increased peripheral pressure due to overload
b. Decreased intravascular fluid volume due to decreased intake
c. Decreased ejection fraction due to acute decompensated heart failure -> acute
decompensated harusnya ekstremitas masih well-perfused
d. Intractable generalized vasodilatation due vasoplegia
e. Increased capillary permeability due to endothelial injury

25
26. A 45-year-old man comes to emergency room with complains of shortness of breath since 1 hour
ago. His complain is getting worse by lying flat. He seems anxious, pale and heavily sweating. He
also complains coughing up blood with pink and frothy sputum. Examination reveals BP 90/50
mmHg, HR 110 x/min, T 37.9oC, O2sat 86% on room air. Chest auscultation reveals rales and ronchi
bilaterally especially on the base of the lungs. From the case above which of the following
treatment is most likely improving the condition of this patient?
a. 2L normal saline IV via large-bore short needle
b. Captopril
c. Diazepam injection
d. Furosemide -> susp pulmo edema
e. Streptokinase

27. A 56 year-old male with longstanding uncontrolled diabetes was admitted to emergency unit with
decreased consciousness since 2 days ago. He also had wound on the sole of his right foot since 2
weeks ago. On physical examination he was somnolen, with BP 80 mmHg/palpation, pulse 120
bpm, respiratory rate 34 times/minute, temperature 38°C. His extremities were cold and clammy.
There was an ulcer in the sole of his right foot, diameter of 5 cm, dirty with pus and necrotic tissue.
Management to control metabolic condition in this patient:
a. Prevention thrombosis with LMWH or UFH
b. Control of blood glucose with intravenous insulin -> metabolic dia?
c. Control of bleeding with fresh frozen plasma
d. Prevention of hypoxemia with intubation
e. Control of infection with broad spectrum antibiotic such as carbapenem

28. A teenage bicycle rider is hit by a truck traveling at a high rate of speed. In the emergency
department, she is actively bleeding from open fractures of her legs, and has abrasions on her chest
and abdominal wall. Her blood pressure is 80/50 mm Hg, heart rate is 140 beats per minute,
respiratory rate is 8 breaths per minute, and GCS score is 6. The first step in managing this patient is
to:
a. Obtain a lateral cervical spine x-ray
b. Insert a central venous pressure line
c. Perform endotracheal intubation and ventilation -> GCS sudah <6 + dia RR rendah bgt?
d. Administer 2 liters of crystalloid solution
e. Apply the PASG and inflate the leg compartments

29. A 55 year-old, right-handed man with long standing history of cigarette smoking, hypertension and
atrial fibrillation was brought to the Emergency Room with sudden onset of confusion and right
sided weakness that started 30 minutes prior to his arrival. On examination; his blood pressure was
180/100 mm mercury, his heart was irregularly irregular, he was awake but unable to follow any
command. He was talking non-sense. Dense right hemi paresis was present. What is the most likely
diagnosis?
a. Left frontotemporal infarction due to occlusion of left middle cerebral artery -> kating
b. Left basal ganglia hypertensive hemorrhage with mass effect and midline shift
c. Left sided subdural hematoma due to un-witnessed trauma
26
d. Complicated migraine attack with prolonged symptoms of aura
e. Brainstem infarction due to vertebral artery thrombosis

30. A 16 year old woman came to your clinic due to vaginal bleeding. Her last menstrual period was 1
month ago, but forgot the exact date. She said the last period was abnormal since only spotting for
few days. She denied sexual contact. She felt a sharp lower abdominal pain, on and off. There was
no muscular defense, only pain on deep palpation. What do you think need to be done in this
patient?
a. Quantitative B-Hcg
b. Abdominal Ultrasound -> kating
c. Qualitative B-Hcg
d. Abdominal MRI
e. Abdominal CT Scan

31. Yuli was harshly treated by her husband from the beginning of her marriage, about three years ago.
Quarrel after quarrel they went through at their rented house in Jakarta, often accompanied by
harsh words and often beaten, until finally once Yuli suffered the miscarriage of her first child. Her
husband later apologized and promised to change his habits. The above events are:
a. Emotional Abuse, Drug Abuse, Verbal Abuse
b. Verbal Abuse Only
c. Physical Abuse, Verbal Abuse
d. Physical Abuse, Emotional Abuse, Verbal Abuse -> consistent pattern of abusive words &
bullying behaviour udah bikin emotional abuse
e. Economic Abuse, Emotional Abuse, Verbal Abuse

32. 50-year-old woman with a history of high blood pressure comes to the emergency department
because of a severe, throbbing right-sided headache associated with nausea, vomiting and
photophobia. The headache began as a dull ache and gradually increased in severity over several
hours. One hour earlier, her left side of the body became paralyzed which remains present. Her
temperature is 37.2oC and there is no resistance on passive flexion of the head. Which of the
following is the most likely diagnosis?
a. Intracerebral hemorrhage
b. Migraine
c. Subarachnoid hemorrhage
d. Brain abscess
e. Temporal arteritis

33. A 28 year-old man is brought to the Emergency Department by ambulance following a generalized
seizure (which has been witnessed by a competent healthcare professional). He does not have
epilepsy and has never had a seizure before. The patient has now recovered and his GCS is 15.
What feature would indicate that the patient should have an urgent CT head scan?
a. Documented hypoglycaemia
b. Aspirin therapy
27
c. Focal neurological deficit -> new onset + penurunan kesadaran + ada severe progressive
headaches
d. Family history of epilepsy
e. Alcoholism

34. Patient came to emergency room due to profuse vaginal bleeding since yesterday. She claimed that
she is pregnant for 3 months. She has just passed some tissues prior to consult and after that the
bleeding diminished. She is on stable vital signs. On physical examination, the abdomen was soft
and non-tender. The cervix was soft and closed, with minimal bleeding from vagina. Ultrasound has
been done and the result is an empty uterus and both adnexas within normal limits. What do you
think happen in this patient and what is the treatment?
a. Incomplete abortion and curettage
b. Complete abortion and observation
c. Complete abortion and curettage
d. Missed abortion and observation
e. Incomplete abortion and observation

35. A female 29 year old patient came to the Emergency Department with generalized tonic clonic
seizure that lasted for 2 minutes. What will be your first drug of choice treatment for this patient?
a. Carbamazepine intravenously
b. Propofol intravenously
c. Phenytoin intravenously -> kating
d. Diazepam intravenously
e. Phenobarbital intravenously

36. A 37 year old man falls 4 meters off of a roof and fractures his cervical spine, causing damage at the
C4 level. He is initially a flaccid quadriplegic with areflexia. This areflexia and flaccidity evolve into
hyperreflexia and spasticity within 2 weeks later. What is the cause of this phenomenon?
a. Spinal cord contusion
b. Spinal shock -> awalnya LMN baru kemudian UMN
c. Spinal cord laceration
d. Spinal cord compression
e. Spinal cord hypoxic

37. A 18 year old patient has brought to Emergency Room came due to abdominal pain. She had just
done an induced abortion in traditional birth attendant 1 hour ago and suddenly the patient felt
severe pain. Her boyfriend asked her to go there to terminate the pregnancy. It was about 2
months pregnancy according to her boyfriend, they had ultrasound before in obstetrician clinic.
Patient is in stage 3 of hypovolemic shock. What do you think happen in this patient and what is the
treatment?
a. Abdominal perforation and exploratory laparotomy
b. Molar pregnancy and suction curettage
c. Ectopic pregnancy and exploratory laparotomy
28
d. Uterine perforation and exploratory laparotomy -> dia ada nyeri
e. Incomplete abortion and curettage -> harusnya tetap ada vaginal bleeding?

38. A 24 year old woman (gravida 2, para 0, abortus 1) is seen in the emergency department because of
vaginal bleeding, and abdominal cramps. Her last menstrual periode was 10 weeks ago. History is
unclear except for an induced abortion. Physical examination reveals a BP of 110/ 70 mmHg, pulse
12, and temperature 38.5 C. The abdomen is tender with slight rebound in the lower quadrants.
The pelvic examination reveals blood in the vault and a foul smelling discharge from the cervix,
which is dilated to 1 cm. The uterus is 8 weeks size and tender. What is the most likely diagnosis?
a. Induced abortion
b. Pelvic inflammatory disease
c. Ectopic pregnancy
d. Incomplete abortion
e. Septic abortion -> soalnya ada discharge bau

39. Suppose in the emergency room, his skull X ray shows linier fracture at the left temporal region,
and the physician decided to do observation for 6 hours, but after 2 hours he became unconscious
again. Now his BP increase to 150/95 mmHg, pulse up to 100 x/minutes, respiratory rate up to 20
x/minutes. The pupil on the left 4 mm/ very slow response to the light, and right pupil 2 mm
response to light. His left arm and leg are inactive compare to the right side. What do you think the
most probable cause of his unconsciousness?
a. There is a hematoma that cause compression of left brainstem -> herniasi
b. There is a secondary brain injury due to prolonged hypoxia
c. The brainstem itself is swelling
d. The brain is swelling and compressing the brainstem brainstem
e. There is a bleeding in the pons

40. A 56 year-old male with longstanding uncontrolled diabetes was admitted to emergency unit with
decreased consciousness since 2 days ago. He also had wound on the sole of his right foot since 2
weeks ago. On physical examination he was somnolen, with BP 80 mmHg/palpation, pulse 120
bpm, respiratory rate 34 times/minute, temperature 38°C. His extremities were cold and clammy.
There was an ulcer in the sole of his right foot, diameter of 5 cm, dirty with pus and necrotic tissue.
What primary management for the latest condition?
a. Norepinephrin 0.01 mg/kg BW/ minute
b. Intubation of endotracheal tube and put on ventilator with control mode -> ini duluuu?
c. Send to surgeon for exploratory laparotomy
d. Coloid loading of 500 ml
e. Crystaloid loading 30 ml/kg BW? -> ini?

41. A 29 year old woman presents with 3 days of dark spotting and mild cramping at 9 weeks gestation.
An ultrasound notes a viable 9 weeks gestation with no obvious problems in the uterus or ovaries.
On obstetrical examination, the cervix is closed, smooth with minimal bleeding through the ostium.
What is the most likely diagnosis?
29
a. Imminen abortion -> mild cramp/ga ada + os servix tertutup + usia kehamilan sesuai + no
expulsion jaringan
b. Inevitable abortion
c. Premature labor
d. Threatened abortion -> ostium tertutup + ga ada tanda jaringan mau keluar + sebagai tanda
adanya kemungkinan aborsi + nyeri ringan bgt + masih ada janin dalam ultrasound
e. Incomplete abortion

42. Patient trauma victim bleeds heavily. The patient getting confused and the blood pressure dropped
to 80/30 mmHg. When attached to monitor, it reveals that the heart rate is 140 bpm, RR 28x/min
and saturation 94%. UO < 0,5 cc/kg/hr. Hb of the patient 6,2 g/dL. You can assume that that patient
is on:
a. Haemorrhagic shock class 3
b. Haemorrhagic shock class 2
c. Haemorrhagic shock class 4
d. Haemorrhagic shock class 1
e. Shock non haemorrhagic

43. A 72 year-old man presents to the emergency department with left-sided weakness and slurred
speech. The weakness appeared suddenly 1 hour earlier while he was brushing his teeth. He has
diabetes mellitus type 2, treated with rosiglitazone and metformin, and he is on warfarin for atrial
fibrillation. He denies word-finding difficulties, headache and abnormal sensations. Blood pressure
is 180/90 mmHg and heart rate is 66/min, irregular. He has left-sided weakness, predominantly of
the arm and face, and a pronator drift on the left side. A bruit is heard over the right carotid artery.
INR is 1.8. Random blood glucose is 125 mg%. Which one of the following is the most useful
imaging study in the initial management?
a. MR angiography
b. Duplex ultrasound studies of carotid arteries?
c. Non-contrast head CT -> ini primary modality untuk initial evaluation pasien suspected stroke
d. Cerebral angiography -> susp stroke hemorrhagic karena ada penggunaan warfarin
e. Transoesophageal echocardiography

44. An acutely anxious 24-year-old woman presents to the Emergency Room with a four hours history
of wheezing and shortness of breath. Arterial blood gases show PaO2 48 mmHg (80-100), PaCO2
31.5 mmHg (35-45), pH 7.5 (7.35-7.45). She is given high flow oxygen by mask and her PaO2 rises to
67 mmHg. What is the most appropriate next therapy?
a. Magnesium infusion
b. Salbutamol nebulizer
c. Aminophylline infusion
d. Ipratropium nebulizer
e. Adrenaline intramuscular

30
45. A 56 year-old male with longstanding uncontrolled diabetes was admitted to emergency unit with
decreased consciousness since 2 days ago. He also had wound on the sole of his right foot since 2
weeks ago. On physical examination he was somnolen, with BP 80 mmHg/palpation, pulse 120
bpm, respiratory rate 34 times/minute, temperature 38°C. His extremities were cold and clammy.
There was an ulcer in the sole of his right foot, diameter of 5 cm, dirty with pus and necrotic tissue.
The primary cause of his vasculatory shock was
a. Alergic reaction that caused vasodilation
b. Unability of the heart to pump blood due to longstanding diabetes and coronary arterial
disease
c. General vasodilatation that leads to vasoplegia
d. Decreased intake due to decreased consciousness
e. Movement of intravascular fluid due to increased permeability caused by inflammation

46. A 78 years old male is admitted to ER with chief complaint severe whole abdominal pain, starting
since 3 days ago. At first he complained of colicky severe abdominal pain, accompanied with nausea
and vomiting, and cannot passage gas, also distended abdomen since 6 days ago. He denies any
fever, but his micturition is decreased since 3 days ago. Previously he had alteration of his bowel
habit since 6 months ago, the stool caliber is decreased and accompanied with fresh blood, mixed
in his feces. No history of chronic diarrhea. Decreased of weight also noticed by the patient and the
family but they don’t know about the number. His physical exam: He is lethargic, anemic, look
cachexic. BP 110/60 mmHg, HR 100x/m, weak but regular. RR 28x/m, Temp 36.8 C. Body weight 60
kg, Height 170 cm. urine output 30 cc/hour. His abdomen: distended, no bowel sound, percussion:
pain at whole abdomen, palpation: tenderness and rebound tenderness at whole abdomen,
muscular rigidity at whole abdomen. His Lab came with result: Hb 10.8gr/dL, HT 32 %, WBC
25.000/mm3, Platelet 350.000/mm3. SGOT SGPT within normal limit, glucose at random 100 gr/dl,
Albumin 2.0 gr/dL. What is another URGENT lab exam you should check for the patient?
a. Alkali fosfatase
b. Ureum and creatinine
c. CEA -> susp kanker colon
d. Anti Hbs Ag
e. Gamma globulin

47. A 32-year-old woman presents with a grade 2 open midshaft femoral shaft fracture as the result of
a high-speed motor vehicle collision. Concomitant injuries include a high-grade splenic laceration
requiring splenectomy as well as a subdural hematoma that requires monitoring and maintenance
of cerebral perfusion pressure. Which of the following is the earliest appropriate management of
the femoral shaft fracture at this time?
a. Irrigation and debridement of the open fracture continued with external fixation
b. Not started any treatment for the fracture until spleen laceration and subdural hematoma are
healed
c. Primary wound closure and immobilization -> kating
d. Immediate ORIF with plate and screw
e. Performed immidiate surgical amputation to minimized blood loss

31
48. Suppose in the emergency room, his skull X ray shows linier fracture at the left temporal region,
and the physician decided to do observation for 6 hours, but after 2 hours he became unconscious
again. Now his BP increase to 150/95 mmHg, pulse up to 100 x/minutes, respiratory rate up to 20
x/minutes. The pupil on the left 4 mm/ very slow response to the light, and right pupil 2 mm
response to light. His left arm and leg are inactive compare to the right side. Suppose the patient
GCS now is 10, with all the result and clinical signs. What should be done for him and how about his
prognosis?
a. Immediately send the patient to the ICU and put him on sedation, and the prognosis is good
b. No need to worry, it’s temporary and the prognosis is good
c. Immediately remove the swelling brain and the prognosis is bad
d. Cannot do anything because the brain is already herniated
e. Immediately remove the blood clot and the prognosis is good

49. A 78 years old male is admitted to ER with chief complaint severe whole abdominal pain, starting
since 3 days ago. At first he complained of colicky severe abdominal pain, accompanied with nausea
and vomiting, and cannot passage gas, also distended abdomen since 6 days ago. He denies any
fever, but his micturition is decreased since 3 days ago. Previously he had alteration of his bowel
habit since 6 months ago, the stool caliber is decreased and accompanied with fresh blood, mixed
in his feces. No history of chronic diarrhea. Decreased of weight also noticed by the patient and the
family but they don’t know about the number. His physical exam: He is lethargic, anemic, look
cachexic. BP 110/60 mmHg, HR 100x/m, weak but regular. RR 28x/m, Temp 36.8 C. Body weight 60
kg, Height 170 cm. urine output 30 cc/hour. His abdomen: distended, no bowel sound, percussion:
pain at whole abdomen, palpation: tenderness and rebound tenderness at whole abdomen,
muscular rigidity at whole abdomen. What is likely to be the cause of the case above?
a. Perforated left colon tumor
b. Perforated left colon due to invagination
c. Perforated splenic abscess
d. Obstructed sigmoid due to volvulus
e. Perforated gastric ulcer to Helicobacter pylori infection

50. A 42-year-old man was send by policeman to emergency department. When arrived at the hospital
he was anxious, there is massive pool of blood around his right femur. His airway and breathing
were clear, his vital signs: BP 80/60 mmHg, HR 120 beat/min, RR 28 breath/min. You also observe
that right femur is edematous, with significant deformity is seen. He still has good distal capillary
refill and good pulsation of right dorsalispedis artery. After few hours, the patient still alert with HR
100x/m, complained of severe pain and the femoral region looks more swollen. You observe that
there is decreased pulse in the right popliteal artery, the skin of the right femur is pale and cold.
What is the possible cause?
a. The patient is having open fracture grade IIIc at the left femur
b. The patient is having femoral fracture, and the segment become more displaced because of
the patient’s movement and now the segment of fracture contused the femoral artery, causing
injury to the femoral artery
c. He is having acute compartement syndrome in the right femoral region which compressed the
right popliteal artery
d. He is having septic shock due to the open wound at the right thigh
32
e. The patient was in haemorrhagic shock, so it is possible that he is in transient response of the
haemorrhagic shock

51. A man, 28 years-old, during hospitalization after severe head injury, had the repetition of his left
arm jerking. According to his mother, it was his first attack and he has no history of other illness.
What is your working diagnosis of this attack?
a. Status convulsivus
b. Epilepsy
c. Pseudoseizure
d. Movement disorder
e. Acute symptomatic seizures

52. A 78 years old male is admitted to ER with chief complaint severe whole abdominal pain, starting
since 3 days ago. At first he complained of colicky severe abdominal pain, accompanied with nausea
and vomiting, and cannot passage gas, also distended abdomen since 6 days ago. He denies any
fever, but his micturition is decreased since 3 days ago. Previously he had alteration of his bowel
habit since 6 months ago, the stool caliber is decreased and accompanied with fresh blood, mixed
in his feces. No history of chronic diarrhea. Decreased of weight also noticed by the patient and the
family but they don’t know about the number. His physical exam: He is lethargic, anemic, look
cachexic. BP 110/60 mmHg, HR 100x/m, weak but regular. RR 28x/m, Temp 36.8 C. Body weight 60
kg, Height 170 cm. urine output 30 cc/hour. His abdomen: distended, no bowel sound, percussion:
pain at whole abdomen, palpation: tenderness and rebound tenderness at whole abdomen,
muscular rigidity at whole abdomen. What has happened to this patient?
a. It is an obstructed bowel case that lead to strangulated ileus and peritonitis in the end
b. It is an obstructed bowel because he has no passage of the stool for 6 days
c. It is a paralytic ileus because the patient has no bowel sound
d. It is an acute abdomen with shock
e. It is a peritonitis because the patient has distended abdomen

53. A 78 years old male is admitted to ER with chief complaint severe whole abdominal pain, starting
since 3 days ago. At first he complained of colicky severe abdominal pain, accompanied with nausea
and vomiting, and cannot passage gas, also distended abdomen since 6 days ago. He denies any
fever, but his micturition is decreased since 3 days ago. Previously he had alteration of his bowel
habit since 6 months ago, the stool caliber is decreased and accompanied with fresh blood, mixed
in his feces. No history of chronic diarrhea. Decreased of weight also noticed by the patient and the
family but they don’t know about the number. His physical exam: He is lethargic, anemic, look
cachexic. BP 110/60 mmHg, HR 100x/m, weak but regular. RR 28x/m, Temp 36.8 C. Body weight 60
kg, Height 170 cm. urine output 30 cc/hour. His abdomen: distended, no bowel sound, percussion:
pain at whole abdomen, palpation: tenderness and rebound tenderness at whole abdomen,
muscular rigidity at whole abdomen. After two hours in the ER while waiting for abdominal imaging
and lab result to be done (and despite all the treatment that you give), the patient is deteriorating:
He is now somnolent, GCS E3M6V5=14, BP 80/60 mmHg, HR 120x/m, RR 36x/m, Temp 37.0 C. What
is likely to be the cause?
a. Pulmonary emboli
33
b. Perforated small bowel
c. Diabetic ketosis
d. CVD
e. Septic shock

54. An 18-year-old adolescent girl with a history of depression and opioid dependence presented to the
emergency department after her father found her lying on the floor unresponsive with shallow
respirations. On admission to the emergency department, the patient had a blood pressure of
95/70 mmHg, heart rate of 50, oxygen saturation was 84% on 100% nonrebreather mask, and
respiration rate was 8 breaths/min. Physical examination showed pinpoint pupils, weight: 66 kg.
Patient is known to have a routine prescription of hydrocodone since she had a car accident last
year. You consider this patient as having an opioid overdose and aim to administer initial dose of
naloxone per intravena. How much naloxone should be administered to this patient?
a. 5.2 mg
b. 4.8 mg
c. 2.4 mg -> paling kecil ini
d. 8.0 mg
e. 6.6 mg

55. A-21 year old medical student presents at the A&E unit Siloam Hospital with yellowing of skin &
eyeballs, tenderness of upper right abdomen, abdominal swelling, nausea-vomiting. She’s also
looks like having general sense of feeling unwell, disorientation and confusion. These symptoms
developed rapidly after she’s ingested around 20 tabs of a drug yesterday. The physician who
examine her thought that she had ingested a drug which it can make a formation of a toxic
metabolite can cause hepatocytes damage. Which of the following is the most appropriate agent
should be given to the patient condition?
a. Epinephrine
b. Atropine
c. Curcuminoid
d. N-acetylcysteine -> pelajar biasanya pake amphetamines
e. Methylprednisolone

56. A 24-year-old boxer was brought to emergency room with unconsciousness. Based on paramedic’s
story he was knocked out during a match after getting hit on his temple. He was initially
unconscious but regains his consciousness several minutes later. On the way to the hospital there
was traffic jam, and he appears to be drowsier until he gives no response. On examination you
found under pain stimuli his eyes are not reacting, he produce incomprehensible sounds,
withdrawal reflex from the stimuli and unequal pupil. What is the most likely diagnosis of this
patient?
a. Intracerebral hemorrhage
b. Subdural hemorrhage
c. Subarachnoid hemorrhage
d. Epidural hemorrhage
e. Brain contusion
34
57. A 26-year-old woman present to Emergency Department after giving birth to a healthy 2800 gr
baby with help from Traditional birth attendance/dukun yesterday. Her family says that after the
baby was delivered, it took nearly 1 hour until the placenta is delivered. She is drowsy and less
responsive, BP 60/40 mmHg, HR 136, RR 28 x/min, with cool and clammy skin. She is brought
because there is a continuous trickle of blood from her vaginal. Which of the following is the most
appropriate management to treat the cause of her condition?
a. Bimanual massage -> atau ini dulu?
b. Apply balloon condom
c. Perform manual placenta/curettage -> kalau placenta lahir lama = masalah jaringan?
d. Misoprostol and oxytocin injection
e. Suture the laceration

58. A male 65 year old patient came to the Emergency Department with seizures starting from the right
extremities then continue to all four extremities, his eyes and head turned to the right side and he
was non responsive during his seizure, which lasted for 3 minutes. After that, he regained full
consciousness. He had a history of stroke with right hemiparesis since 2 years ago. EEG revealed
abnormality over the left temporal region. What would be the best diagnosis for this patient?
a. Status epilepticus
b. Epilepsy focal onset to bilateral tonic clonic with impaired awareness
c. Acute symptomatic seizure
d. Epilepsy focal onset with impaired awareness
e. Epilepsy generalized onset

59. A 35-year-old woman come to the emergency room with complains of chest pain since 3 hour ago.
She said that the pain is not radiating, sharp on her left chest and felt worse every time she inhaled.
The pain does not alleviated by resting, nor aggravated by exercise. She also had complains of
productive cough over 1 week. She had routinely taking oral contraception. She is a business
woman who just travels from New York to Singapore in a long range flight. Examination reveals BP
130/90 mmHg, HR 120 x/min, RR 32 x/min, T 38oC. What is the definitive treatment?
a. Propranolol
b. Amlodipine
c. Oxygen 100% via Nasal prongs 2 L/min
d. Aspirin
e. Low-molecular weight heparin

60. A young adult, 16 year-old is watching tv-program, suddenly stops her activity, unresponsive, then
falls down and starts convulsion. She had previous same attacks, twice in a month since two
months before. When there is no attack, she is normal. What is the working diagnosed of this case?
a. Acute symptomatic seizure
b. Status epilepticus
c. Epilepsy
d. Status convulsivus
35
e. Behavior disorder

61. A 62 –year-old man come to Emergency Department with chief complaint a sudden severe chest
pain 4 hours ago. The substernal chest pain start suddenly while he was watching his favourite
football match on TV. The pain lasted for about 30 minutes, radiated to his jaw and left arm while
he sweated a lot. His wife force him to go seek medical attention although now he does not feel the
pain anymore. He has a history of abnormal fasting glucose test but not take any medicine
regularly. You perform an ECG exam, and order cardiac enzyme. The ECG report shows a sinus
rhythm ECG and no ST segment deviation. What would you do now?
a. Consult Cardiologist for primary percutaneous coronary intervention
b. Allow the patient to go home and ask him to come for check up tomorrow
c. Perform CT scan to find another diagnosis
d. Observe the patient while waiting for cardiac enzyme -> tunggu enzim jantung buat bedain dia
UAP/NSTEMI
e. Consult Cardiologist for emergency echocardiography

62. Which of these supporting examinations is considered an adjunct to the primary survey in
managing a trauma case?
a. None of the above -> utamanya ECG + pulse oximetry + CO 2 monitoring + assessment
ventilatory rate + ABG + bisa urinary catheter untuk asses UO & hematuria + bisa gastric
cathehter untuk dekompresi distensi
b. Abdominal x-ray
c. Chest CT-scan
d. Pelvic x-ray
e. Lower extremity x-ray

63. A 30 year old man injured his thoracic spine in motor vehicle accident 2 years ago. Now he still has
pain and thermal sensation loss on part of his left body and propioception loss in his right foot.
There is still a paralysis of the right lower extremity as well. This patient most likely has which of the
following spinal cord conditions?
a. Complete transection
b. Hemisection syndrome
c. Anterior cord syndrome
d. Central cord syndrome
e. Posterior collum syndrome

64. A 42-year-old man was send by policeman to emergency department. When arrived at the hospital
he was anxious, there is massive pool of blood around his right femur. His airway and breathing
were clear, his vital signs: BP 80/60 mmHg, HR 120 beat/min, RR 28 breath/min. What stages of
hemorrhagic shock in this patient?
a. Class V
b. Class I
c. Class IV
36
d. Class II -> kating
e. Class III

65. A 19-year-old woman presents with feses came out from her vagina. She has just delivered a 3800 g
infant vaginally after a 15 hour labor assisted by traditional birth attendance/dukun 1 week ago.
The delivery itself was spontaneous after 5 minutes of pushing with no proper maneuver, the
patient said that there is tear at her vagina but the traditional birth attendance didn’t do anything
for the tear. What is the most likely cause of her complaints?
a. Perineal tear 4th degree
b. Perineal tear 1st degree
c. Perineal tear 5th degree
d. Perineal tear 3rd degree
e. Perineal tear 2nd degree

66. A 70 y.o. female is admitted to ER at 11 PM with chief complain diffuse/whole severe abdominal. At
first the pain was only in her epigastric area for 4 days and then it spread to whole abdomen for the
last 2 days. She also have continuous progressive fever since this morning, with very minimal oral
intake since this morning also. Since 3 days ago she has melena, without diarrhea, with normal
stool volume and frequency. She also has decreased micturition. No history of abdominal trauma,
no history of previous episode like this. She has history of chronic gastritis and chronic knee
arthritis. She routinely consumed painkiller for her knee arthritis, bought by the patient herself
(self-medication) for more than 10 years. She now looks sleepy and lethargic with GCS E3M6V5 BP
100/60 mmHg, HR 120x/m, RR 20x/m, Temp 38C. Skin is dry with decreased turgor. Extremity is
cool, clammy with normal capillary refill. Abdomen is distended, negative liver dullness, decreased
bowel sound, muscular rigidity is present with tenderness and rebound tenderness at palpation.
Digital Rectal Examination/DRE shows that she has decreased sphincter tone, smooth mucosa,
rectal vault is normal with tenderness all direction. The punctum maximum of pain is at the
epigastric region. At your gloves after DRE you found black bloody stool with foully smelling. Now
for source control and the definitive treatment, what will you do?
a. Broad spectrum antibiotics for infection control
b. Consult the surgeon for emergency exploratory laparotomy
c. Emergency upper GI endoscopy for bleeding control
d. Emergency blood and bleeding factor transfusion
e. Emergency hemodialysis

67. A 30-year-old woman is brought into your clinic with burns affecting both of her entire upper
extremities and the anterior surfaces of both legs after spilling boiling soup from the stove while
cooking approximately two hour ago. Her weight is 48 kg, height 150 cm, blood pressure 90/60
mmHg, heart rate 100 bpm, respiratory rate 20 x/min. Using Parkland formula how much fluid you
need to administer EACH hour for the next 6 hour?
a. 378 cc
b. 824 cc
c. 132 cc
d. 3456 cc
37
e. 576 cc -> tapi harusnya dikasih dalam 8 jam?

68. A female 29 year old patient came to the Emergency Department with generalized tonic clonic
seizure that lasted for 2 minutes. The seizure continued for 3 times with no regain of consciousness
between seizures. What would be your diagnosis now?
a. Epilepsy focal with impaired awareness
b. Status epilepticus
c. Epilepsy focal to bilateral tonic clonic with impaired consiousness
d. Epilepsy generalized onset
e. Acute symptomatic seizure

69. A male, 66 years old came to ER with shortness of breath since a week before. He had chronic
cough as he has history of smoking since age of 20. He had diabetes since 5 years ago and was
treated with metformin 500 mg t.i.d., and he also had history of shortness of breath due to chronic
heart failure treated with furosemide 40 mg once daily. On physical examination, his blood pressure
was 80 mmHg/palpation, heart rate 130 beats per minute, respiratory rate 40 times per minute and
temperature 36.8°C. On cardiac examination there were gallop with normal heart sound. Her
extremities were edematous and cyanotic. Laboratory revealed hemoglobin level 16.5 g/dL,
leucocyte 12,000/μL, blood glucose 182 mg/dL. Radiology: cardiothoracic ratio was 55%, with
increased intercostal space. What is the mechanism of current problem happening in the case
above?
a. Decreased ejection fraction due to acute decompensated heart failure -> ?
b. Decreased intravascular fluid volume due to decreased intake
c. Decreased preload to the left ventricle due to pulmonary hypertension
d. Intractable generalized vasodilatation due vasoplegia
e. Increased capillary permeability due to endothelial injury

70. A girl just quarrel with her boyfriend because She unintentional looked message in an intimate
word and read repeatedly. It’s sender from the other girl. After incident, she try suicide as a cry for
help, to get attention, unusual but not fatal. What therapy gave properly?
a. Group therapy
b. Psychotherapy
c. Hypnotherapy
d. Social therapy
e. Pharmacotherapy

71. A 10-month-old boy presents to the ED with a 1-day history of fever to 40°C, increased irritability,
decreased breast-feeding, and refusal of solid foods. The parents brought him in after two 30-
second episodes of generalized jerking that occurred over a 20-minute span. Your examination
reveals an awake but lethargic infant. The anterior fontanelle is flat, the tympanic membranes and
oropharynx are moist and not erythematous, the lungs are clear, and the heart and abdominal
examinations are normal. He has no focal neurologic findings. Which of the following is the best
next step in management?
38
a. Lumbar puncture -> masih usia <10 bulan jadi cek aja takut meningitis
b. Discharge from ED to follow up with his primary care provider in 24 hours
c. Computerized tomography of the head
d. Intravenous antibiotic
e. Admission overnight for observation

72. A male patient, 32 years old had a motorcycle accident and was brought to your emergency
department. Primary survey has been cleared, the patient was in shock with a heart rate of
134x/min and blood pressure of 80/60. Fluid challenge had already been given (1 L of warm isotonic
solution) and the went up to 100/80 for 15 minutes before falling back to 70/50. What is the most
likely cause?
a. Cervical trauma
b. Occult bleeding
c. Spinal shock
d. Septic shock
e. Cardiogenic shock -> kating

73. A 24 year old woman presents with 3 days of dark spotting and mild cramping at 8 weeks gestation.
An ultrasound notes a gestational sac appropriates of 8 weeks age of gestation, with no yolk sac nor
fetal echo has seen. No mass has seen on both adnexa and the ovaries are normal. On obstetrical
examination, the cervix is closed, smooth with minimal bleeding through the ostium. What is the
most likely diagnosis?
a. Threatened abortion
b. H. mole pregnancy
c. Missed abortion
d. Early pregnancy
e. Blighted ovum

74. A five years-old girl developed a high fever, of 40C, and increasing difficulty breathing within a 10-
hour period. She refused to drink or eat, and she cried only minimally despite her ill-looking
appearance. Her vital status: disoriented, HR 140x/m, RR 48x/m, temp 40C, O2 sat 80%. There is
some drooling of saliva down her chin, and a soft inspiratory stridor is heard. She had no preceding
coryzal symptoms. What is the most appropriate next step of management?
a. Arrange urgent intubation?
b. Give nebulized adrenaline to the patient
c. Give antibiotics
d. Order a chest X ray
e. Take a throat swab for culture and sensitivity

75. You are working in an Emergency Department when a 19 year-old student is brought in by
ambulance after her first generalised tonic clonic seizure. On examination the Glasgow Coma scale
(GCS) is 15, she has a grade 3/5 weakness of the right arm and leg with an upgoing right plantars

39
response. Which of the following statements regarding further management in this case is most
accurate?
a. A lumbar puncture should be performed to exclude meningitis
b. An EEG should be performed urgently
c. A single seizure does not require further investigation at this stage
d. Intravenous diazepam should be administered to prevent a further seizure
e. A CT scan of the brain should be performed urgently -> ada weakness (deficit neurologis?)

76. A 65-year-old gentleman presented to Emergency Department in unconscious condition. His ECG is
attached. (VF) He has no pulse. Which of the following is the most appropriate intervention?
a. Thrombolytic therapy
b. DC shock -> VF shockable
c. Valsava Maneuver
d. Carotid Message
e. Adenosine triphosphate intravenous

77. Female 18 year-old weighted 50 kg, came to the Emergency with chief complain burn at her right
upper extremity and anterior chest and abdomen. She had an accident while carrying a bucket of
hot water. Suppose you as the attending physician in the emergency willing to give IV fluid to the
patient using Parkland formula, how much fluid would you give?
a. 450 mL/h for 16 hours and 225 mL/h for 8 hours
b. 2000 mL for 24 hours
c. 450 mL/h for 8 hours and 225 mL/h for 16 hours
d. 340 mL/h for 8 hours and 170 mL/h for 16 hours
e. 340 mL/h for 16 hours and 170 mL/h for 8 hours

78. I feel like I'd be struck by lightning if I didn't turn three times to the right. Or to believe my sister
was going to have an accident because I didn't stop at the odd-numbered stairs. My feet, my hands,
always don't stop holding things over and over again until they feel right. Often in my heart I count
odd numbers until my feelings become calmer. What disorder is this?
a. Obsessive-Compulsive Disorder -> true obsessions/compulsion yang irrational & berulang
b. Narcissistic Personality Disorder
c. Obsessive-Compulsive Personality Disorder -> ga directed by uncontrollable thought/irrational
jadi lebih perfectionism + long term
d. Avoidant Personality Disorder
e. Schizotypal Personality Disorder -> kayak social anxiety

79. A 78 years old male is admitted to ER with chief complaint severe whole abdominal pain, starting
since 3 days ago. At first he complained of colicky severe abdominal pain, accompanied with nausea
and vomiting, and cannot passage gas, also distended abdomen since 6 days ago. He denies any
fever, but his micturition is decreased since 3 days ago. Previously he had alteration of his bowel
habit since 6 months ago, the stool caliber is decreased and accompanied with fresh blood, mixed
in his feces. No history of chronic diarrhea. Decreased of weight also noticed by the patient and the
40
family but they don’t know about the number. His physical exam: He is lethargic, anemic, look
cachexic. BP 110/60 mmHg, HR 100x/m, weak but regular. RR 28x/m, Temp 36.8 C. Body weight 60
kg, Height 170 cm. urine output 30 cc/hour. His abdomen: distended, no bowel sound, percussion:
pain at whole abdomen, palpation: tenderness and rebound tenderness at whole abdomen,
muscular rigidity at whole abdomen. For the first management, what will you do as the ER doctor?
a. Send the patient for emergency laparotomy
b. Give the patient intravenous albumin
c. NPO and fluid rehydration -> atau ini?
d. Send the patient to ICU since he has sepsis condition
e. Give the patient Packed Red Cell Transfusion

80. A previously healthy 3 year-old child (weight 20 kg) had a single generalised convulsion lasting 5
min last year while he is having high fever. The parent of previous patient brings the patient home
after they meet local doctor after examination for fever accompanied with upper respiratory tract
infection, and then suddenly the child is having a generalized convulsion again at home and brought
to the hospital again. After the convulsion, the child was crying loudly and still active despite she is
still having fever. Now 2 hours later she is in generalized convulsion again (2nd episode of
convulsion for today) while she is arriving at the hospital. What is the first appropriate management
for this patient?
a. Give lorazepam intravenous injection -> kating
b. Give antibiotics because oral fluid therapy is not enough
c. Give paracetamol suppositoria because the patient is in convulsion so she could not take the
oral medication
d. Muscle relaxant intravenous
e. Give diazepam per rectal 10mg

81. A 47 year old male was brought to emergency due to history of brief unconsciousness. The family
told that the patient was suffering from edema in his both leg since a week ago, and he was
prescribed Furosemide for his problem. Physical examination showed normal heart sound and
normal vesicular sound on both lungs. What do you suggest causing the brief unconscious in this
patient?
a. Drug induced orthostatic hypotension -> kating
b. Drug induced pro-arrhythmias
c. Pressure to carotid sinus
d. Stimulation to trigeminal nerve
e. Hyperactivity of vasovagal reflex

82. Mr. Hendry, 34 years old male, got a motor vehicle accident. He was not using helmet when the
accident happened. When he fell, his head hit the road. He become unconscious after that and
never again regain his consciousness until now. When ambulance came, his blood pressure is
120/80 mmHg, pulse 88 x/minutes, respiratory rate was 12 x/ minutes. What do you think the
cause of his unconsciousness?
a. He was drunk before the accident
b. The concussion effect to the brain stem
41
c. Broken bone at his head because he was not using helmet
d. Big hematoma in his brain and compressing his brain stem
e. Other medical problem such as heart attack, hyperglycemia, etc.

83. Mr. Ahmad, 65-year-old, had an anterior myocardial infarction one year ago. He has shortness of
breath and easily fatigue when walking long distance or climbing more than 2 stairs. With regular
daily activity, he has no marked limitation. What is his functional class?
a. NYHA class 4
b. NYHA class 1
c. NYHA class 3
d. Cannot be determined
e. NYHA class 2

84. A 62 –year-old man come to Emergency Department with chief complaint a sudden severe chest
pain 4 hours ago. The substernal chest pain start suddenly while he was watching his favourite
football match on TV. The pain lasted for about 30 minutes, radiated to his jaw and left arm while
he sweated a lot. His wife force him to go seek medical attention although now he does not feel the
pain anymore. He has a history of abnormal fasting glucose test but not take any medicine
regularly. What is the most appropriate diagnosis now?
a. Psychological chest pain
b. Acute pericarditis
c. Aortic regurgitation
d. Acute coronary syndrome -> biasanya substernal pain + last lumayan lama + severe pain
e. Mitral Stenosis

85. A 30-year-old man is struck by a car traveling at 56 kph (35 mph). He has obvious fractures of the
left tibia near the knee, pain in the pelvic area, and severe dypsnea. His heart rate is 180 beats per
minute, and his respiratory rate is 48 breaths per minute with no breath sounds heard in the left
chest. A tension pneumothorax is relieved by immediate needle decompression and tube
thoracostomy. Subsequently, his heart rate decreases to 140 beats per minute, his respiratory rate
decreases to 36 breaths per minute, and his blood pressure is 80/50 mmHg. Warmed Ringer's
lactate is administered intravenously. The next priority should be to:
a. Perform diagnostic peritoneal lavage or abdominal ultrasound
b. Perform arterial embolization of the pelvic vessels
c. Obtain abdominal and pelvic CT scans -> ini?
d. Perform external fixation of the pelvis -> kating
e. Perform a urethrogram and cystogram

86. At 3 AM a middle-aged man arrives at the Accident and Emergency Department, escorted by police.
He sings at the top of his lungs and throws kisses at everyone. His wife called the police because he
has been singing and shouting at her and at neighbours since 10 PM. She says that he stopped his
medication two weeks earlier because he felt he was full of energy and needs no medication. What
is the most likely diagnosis?
42
a. Heroin intoxication
b. Alcoholic intoxication
c. Acute mania
d. Cyclothymia
e. Schizophrenic disorder

87. What should a three years-old boy with three episodes of febrile seizure within the past year best
be treated with?
a. No medication -> kalau usia <6 bulan atau kejang 4x dalam 1 tahun baru dikasih anticonvulsant
b. Phenobarbital
c. Valproic acid
d. Carbamazepine
e. Diazepam

88. A girl just quarrel with her boyfriend because She unintentional looked message in an intimate
word and read repeatedly. It’s sender from the other girl. After incident, she try suicide as a cry for
help, to get attention, unusual but not fatal. What’s kind of suicide this girl act?
a. Suicide gesture
b. Suicide attempt
c. Suicide equivalent
d. Suicide gamble
e. Suicide serious

89. An 18-year-old adolescent girl with a history of depression and opioid dependence presented to the
emergency department after her father found her lying on the floor unresponsive with shallow
respirations. As an on duty doctor in emergency department, what is your first action?
a. Obtain history from the family of the patient
b. Apply the monitors
c. Take blood and urine toxicology test
d. Administer naloxone -> kating
e. Intubate the patient -> stabilkan pasien dulu baru kasih naloxone

90. A 14 year-old female was diagnosed as epileptic (tonic-clonic type) and has been treated with
carbamazepine. She has been poorly compliant with her medications, and has been rushed to the
A/E with status epilepticus. Which ONE of the following is the MOST APPROPRIATE medication for
IMMEDIATE control of her convulsions?
a. Phenytoin, i.v. infusion -> kating
b. Diazepam, i.v. injection
c. d-tubocurarine, i.v. injection
d. Phenobarbitone sodium, i.v. injection
e. Thiopentone sodium, i.v. bolus injection

43
91. A 30 years old male come to ER at 10 AM with chief complaint of 2nd degree burn injury at all of his
left hand, left forearm(all circumference) and left elbow joint (all circumference) due to scald burn
injury/ hot water accident, happened 2 hours ago. The pic is shown below: How many percent is
the burn injury?
a. 30%
b. 18.5%
c. 20%
d. 6% -> kating
e. 25%

92. A 68 years old male brought to emergency department due to recurrent brief episode of
unconsciousness in the past 6 months. He had history of hypertension and diabetes mellitus. The
doctors then perform tilt testing, and the result showed blood pressure drop and bradycardia.
What is your diagnosis in this patient?
a. Situational syncope
b. Orthostatic hypotension syncope
c. Vascular steal syndrome
d. Carotid sinus syncope
e. Vasovagal syncope

93. A 78 years old female is admitted to ER because she has difficulty in breathing. She is a diabetic
mellitus type 2 patient, she took her diabetic medicine on and off. She has history of difficulty of
breathing since 1 week ago, starting with mild cough with thick cloudy sputum and continuous
fever, treated with antibiotics from local doctor. She is now alert, agitated breathing heavily with
could speak one or two words only, with BP 110/60 mmHg, HR 110x/m, regular and equal pulse and
warm extremities. RR 36x/m, Temp 38.7 C, O2 Sat 92 % with room temperature. After 30 minutes
observation and the treatments, the lab come to you: Hb 10gr/dL, Ht 30, WBC 22.000/mm3,
Platelet 325.000/mm3, Random blood glucose 395 gr/dL, Ureum, creatinine, electrolytes and
SGOT/SGPT are remarkable. Blood gas analysis: pH 7.2 (7.35-7.45), HCO3 25.3 (22-26), PaO2 40
mmHg (80-100), PCO2 45 mmHg (35-45), Base excess -5.2 (-2.0-+2.0), Sat O2 94% (95-100) with 10
liters oxygen using non-rebreathing face mask. BP, HR, RR and Temperature of the patient after 30
minutes is about the same since she is coming (no changing). Thoracic x-ray show that both lungs
shows basal consolidation on the both lungs. What prioritized step should you do next?
a. Give her natrium bicarbonate
b. Give her blood transfusion with packed red cell because she is anemic
c. Intubate her and put her in ventilator
d. Check the blood culture
e. Give her intravenous insulin drip/sliding scale
94. A 36-year-old woman is admitted to company clinic following a workplace accident. She says that
her leg and foot is splashed by a liquid from a falling chemical container. She is in great pain and
crying for your help. What is the first action you take?
a. Contact the plastic surgeon on call urgently
b. Give pain reliever
c. Test the liquid on affected area with litmus paper
d. Commence water irrigation
44
e. Perform a primary survey of the patient

95. A 28-year-old primigravid woman presents to emergency room with a history of 10 weeks
amenorrhoea followed by heavy vaginal bleeding and crampy abdominal pain. On examination the
uterus is bulky and the internal ostium open. Which of the following will you advise the patient?
a. Evacuation of retained products be performed
b. Tocolytic agents orally
c. An ultrasound be performed -> kating
d. A pregnancy test be performed -> buat tau dia hamil atau tidak
e. Antibiotics be taken

96. A 70 y.o. female is admitted to ER at 11 PM with chief complain diffuse/whole severe abdominal. At
first the pain was only in her epigastric area for 4 days and then it spread to whole abdomen for the
last 2 days. She also have continuous progressive fever since this morning, with very minimal oral
intake since this morning also. Since 3 days ago she has melena, without diarrhea, with normal
stool volume and frequency. She also has decreased micturition. No history of abdominal trauma,
no history of previous episode like this. She has history of chronic gastritis and chronic knee
arthritis. She routinely consumed painkiller for her knee arthritis, bought by the patient herself
(self-medication) for more than 10 years. She now looks sleepy and lethargic with GCS E3M6V5 BP
100/60 mmHg, HR 120x/m, RR 20x/m, Temp 38C. Skin is dry with decreased turgor. Extremity is
cool, clammy with normal capillary refill. Abdomen is distended, negative liver dullness, decreased
bowel sound, muscular rigidity is present with tenderness and rebound tenderness at palpation.
Digital Rectal Examination/DRE shows that she has decreased sphincter tone, smooth mucosa,
rectal vault is normal with tenderness all direction. The punctum maximum of pain is at the
epigastric region. At your gloves after DRE you found black bloody stool with foully smelling. You
need to stabilize the patient, what do you do? (prioritize your action)
a. Intubate and send the patient to the ICU
b. Give fluid and blood to the patient, antibiotics and analgetics
c. Emergency consult to surgeon for emergency exploratory laparotomy -> difuse peritonitis
d. Lab check and abdominal xray, abdominal CT if needed
e. Non per oral, Give oxygen and IV Fluids, abdominal decompression, IV antibiotics and analgetic
-> kating

97. Suppose in the emergency room, his skull X ray shows linier fracture at the left temporal region,
and the physician decided to do observation for 6 hours, but after 2 hours he became unconscious
again. Now his BP increase to 150/95 mmHg, pulse up to 100 x/minutes, respiratory rate up to 20
x/minutes. The pupil on the left 4 mm/ very slow response to the light, and right pupil 2 mm
response to light. His left arm and leg are inactive compare to the right side. What do you need to
do to prove your thinking about the above case?
a. Do MRI of the head
b. Do CT scan of the head -> susp hematoma
c. Do a lumbal puncture to see whether the is a blood stained CSF
d. Do another skull X ray to see the changes
e. Do the blood gas analysis
45
98. A 45-year-old man comes to emergency room with complains of shortness of breath since 1 hour
ago. His complain is getting worse by lying flat. He seems anxious, pale and heavily sweating. He
also complains coughing up blood with pink and frothy sputum. Examination reveals BP 90/50
mmHg, HR 110 x/min, T 37.9oC, O2sat 86% on room air. Chest auscultation reveals rales and ronchi
bilaterally especially on the base of the lungs. What is the most likely diagnosis?
a. Acute pulmonary edema
b. Heart failure NYHA grade III
c. Hypovolemic shock
d. Panic attack
e. Acute myocardial infarction

99. Mr. Hendry, 34 years old male, got a motor vehicle accident. He was not using helmet when the
accident happened. When he fell, his head hit the road. He become unconscious after that and
never again regain his consciousness until now. When ambulance came, his blood pressure is
120/80 mmHg, pulse 88 x/minutes, respiratory rate was 12 x/ minutes. Suppose when Mr. Hendry
arrived in the hospitals, he become conscious with a bad headache. He forgot how he fell but he is
fully oriented. No focal neurologic finding. His general condition is unremarkable. What would your
thought about his cortico spinal tract at that moment?
a. Cannot be concluded yet
b. Cannot be assessed clinically
c. It’s intact
d. Probably partial injured
e. It’s injured

100. A-47 year old man with a history of taking 250-300mL wine with high ethanol concentration. He
came to emergency department due to black, bloody stools, and vomit that looks like coffee
grounds after consuming aspirin for around a week. The doctor thought that might be because of
local effect combination of aspirin-alcohol at gastric mucosal. But actually, the problem also occur
when such patients took many other drugs like benzodiazepine (clonazepam, alprazolam). Acute
ethanol ingestion may potentiate the CNS effects of benzodiazepine or tolerance of the drug may
develop with chronic alcohol abuse. Which of the following is the most likely mechanism of alcohol
can make that situations?
a. Decrease intestinal transit time
b. Desensitizes receptors of other drugs
c. Inhibit renal excretion with maximizing other drugs resorption
d. Decrease ability of hepatocyte to metabolize other drugs
e. Binding of other drugs

46
2018
A-21 year old medical student presents at the A&E unit Siloam Hospital with yellowing of skin &
eyeballs, tenderness of upper right abdomen, abdominal swelling, nausea-vomiting. She’s also
looks like having general sense of feeling unwell, disorientation and confusion. These symptoms
developed rapidly after she’s ingested around 20 tabs of a drug yesterday. The physician who
examine her thought that she had ingested a drug which it can make a formation of a toxic
metabolite can cause hepatocytes damage. Which of the following is the most appropriate
agent should be given to the patient condition?
a. N-acetylcysteine
b. Atropine
c. Curcuminoid
d. Methylprednisolone
e. Epinephrine

During a treatment of an overdose case in emergency, breathing begins to slow down and
patient become apneic. The patient become unconscious fall into a coma for more than 5
minutes. After the doctor realized it he/she is trying to give patient treatments. Despite all of
the treatments, the patient still unconscious for the rest of his life (vegetative state). Why do
you think this happened?

a. Prolonged hypooxygenation causing permanent brain damage


b. Brain damage accompanied by liver failure
c. Brain injury caused by multiple treatments given during hypoxemia
d. Multiple organ dysfunction due to drug overdose
e. Severe sepsis causing permanent brain damage

A 45-year-old man comes to emergency room with complains of shortness of breath since 1
hour ago. His complain is getting worse by lying flat. He seems anxious, pale and heavily
sweating. He also complains coughing up blood with pink and frothy sputum. Examination
reveals BP 90/50 mmHg, HR 110 x/min, T 37.9oC, O2sat 86% on room air. Chest auscultation
reveals rales and ronchi bilaterally especially on the base of the lungs. From the case above
which of the following treatment is most likely improving the condition of this patient?

a. Furosemide
b. 2L normal saline IV via large bore short needle
c. Streptokinase
d. Captopril
e. Diazepam injection

A 50 year old man, came to Emergency Department with shortness of breath, cough and fever.
He was diagnosed with Covid-19. He has to be admitted but unfortunately, all hospitals in
Jakarta has been fully occupied. His family decided to bring him home and emergency doctor

47
told them to keep monitoring his saturation with pulse oxymetry. What exactly that the doctor
want to know?

a. Ventilation
b. Restriction
c. Diffusion
d. Perfusion
e. Obstruction

A 44 years old female came to ER with pain on the right upper quadrant of the abdomen since 3
days before. She felt nausea and vomitted 4-5 times per day. She had diabetes, history of gall
bladder stone and chronic heart failure since 5 years ago and she took metformin 2x500 mg,
bisoprolol 1x5 mg, furosemid 1 x 40 mg orally. On physical examination, her blood pressure was
80 mmHg/palpation, heart rate 130 beats perminute, respiratory rate 40 times perminute and
temperature 39°C. Her extremities were clammy and cold. Laboratory revealed hemoglobin
level 11.5 g/dL, leucocyte 18,000/μL, blood glucose 249 mg/dL, blood ketone 0.9 g/dL. What is
the mechanism of current main problem in this patient?
a. Decreased ejection fraction due to chronic heart failure
b. Increased capillary permeability due to endothelial injury
c. Intractable generalized vasodilatation due to vasoplegia
d. Generalized vasodilatation caused by medication with bisoprolol
e. Decreased intravascular fluid volume due to nausea and vomiting

A 37 year old man came to Emergency & Accident Unit with feeling of apprehension, fear and
palpitation, even mounting to feeling of impending death. After being examined by doctor, the
results showed that nothing was wrong. After3 weeks, he got the same feelings, and he had to
go to the hospital again. But this time the Emergency staffs were busy, and he was kept to wait.
Until about 15-20 minutes he had not been attended to, but his fear and palpitation subsided
and became calm by it. What would you think about the suffering that this person had?
According to the above case, what would you prioritize?

a. Plan for psychoanalytical psychotherapy


b. Advise him to go for refreshing
c. Advise him to go for Yoga exercise
d. Give some explanation to the nature of his illness
e. Prescribe drug for his distress

A 65 year old male came to Emergency Department due to shortness of breath since 5 days ago
and getting worse since 1 day before admission. He complained that the dry cough and chest
pain make his shortness of breath worse. He travelled to Surabaya 1 week ago for office matter.
In Emergency Department, he looked somnolen, with vital sign BP 110/70, RR 45
times/minutes, HR 126x/minutes, temp 38.6 ,SpO2 77%. Chest Xray showed crazy paving at

48
both of his lung. BGA showed pH 7,15, pCO2 98, pO2 45, HCO3 32, BE +6, SpO2 77% with NRM
15 lpm. As an emergency doctor, what do you think about current condition of this patient?

a. Acute Lung injury


b. Acute Respiratory Distress Syndrome
c. Highly suggestive severe bacteria pneumonia
d. Highly suggestive of COVID-19
e. Ventilation impairment

Mr. Hendry, 34 years old male, got a motor vehicle accident. He was not using helmet when the
accident happened. When he fell, his head hit the road. He become unconscious after that and
never again regain his consciousness until now. When ambulance came, his blood pressure is
120/80 mmHg, pulse 88 x/minutes, respiratory rate was 12 x/ minutes. What do you think the
cause of his unconsciousness?

a. Broken bone at his head causing infection


b. Big hematoma compressing his scalp
c. Other medical problem such as heart attack, hyperglycemia, etc
d. He was drunk before the accident
e. The concussion effect to the brainstem

Which of these supporting examinations is considered an adjunct to the primary survey in


managing a trauma case?

a. Abdominal x-ray
b. Chest CT scan
c. Lower extremity x-ray
d. Pelvic x-ray
e. None of the above

A 55 year-old, right-handed man with long standing history of cigarette smoking, hypertension
and atrial fibrillation was brought to the Emergency Room with sudden onset of confusion and
right sided weakness that started 30 minutes prior to his arrival. On examination; his blood
pressure was 180/100 mm mercury, his heart was irregularly irregular, he was awake but
unable to follow any command. He was talking non-sense. Dense right hemi paresis was
present. What is the most likely diagnosis?

a. Brainstem infarction due to vertebral artery thrombosis


b. Left basal ganglia hypertensive hemorrhage with mass effect and midline shift
c. Complicated migraine attack with prolonged symptoms of aura
d. Left frontotemporal infarction due to occlusion of left middle cerebral artery
e. Left sided subdural hematoma due to un-witnessed trauma

49
At 3 AM a middle-aged man arrives at the Accident and Emergency Department, escorted by
police. He sings at the top of his lungs and throws kisses at everyone. His wife called the police
because he has been singing and shouting at her and at neighbours since 10 PM. She says that
he stopped his medication two weeks earlier because he felt he was full of energy and needs no
medication. What is the most likely diagnosis?
a. Heroin intoxication
b. Alcoholic intoxication
c. Schizophrenic disorder
d. Cyclothymia
e. Acute mania

You are working in an Emergency Department when a 19 year-old student is brought in by


ambulance after her first generalised tonic clonic seizure. On examination the Glasgow Coma
scale (GCS) is 15, she has a grade 3/5 weakness of the right arm and leg with an upgoing right
plantarsresponse. Which of the following statements regarding further management in this
case is most accurate?
a. A lumbar puncture should be performed to exclude meningitis
b. A CT scan of the brain should be performed urgently
c. A single seizure does not require further investigation at this stage
d. An EEG should be performed urgently
e. Intravenous diazepam should be administered to prevent a further seizure

Female 18 year-old weighted 50 kg, came to the Emergency with chief complain burn at her
right upper extremity and anterior chest and abdomen. She had an accident while carrying a
bucket of hot water. Suppose you as the attending physician in the emergency willing to give IV
fluid to the patient using Parkland formula, how much fluid would you give?
a. 340 mL/h for 16 hours and 170 mL/h for 8 hours
b. 2000 mL for 24 hours
c. 450 mL/h for 16 hours and 225 mL/h for 8 hours
d. 450 mL/h for 8 hours and 225 mL/h for 16 hours
e. 340 mL/h for 8 hours and 170 mL/h for 16 hours

A 2-years-old boy was found unconsciousness in his house. He accidentally consumed morphine
sustained release tablets, which is usually taken by his grandmother who has chronic pain from
osteoarthritis. The patient was rushed into the triage of Emergency Department. As a doctor on
duty, what is the priority management of this patient?
a. Administer Naloxone
b. ABC resuscitation
c. Administer Oxygen
d. Taking medical history and do the physical examination
e. Do the opioid testing

50
A 34 year old female came to your Emergency department with profuse bleeding. She has just
deliver a twin baby half an hour ago weigh 2800 grams and 2600 grams, this is her
6th pregnancy. The patient is unconscious, BP 80/palp, HR 110 beats /min, when you inspect the
vagina, heavy bleeding still occurs. What’s the most common possible cause of the bleeding?
a. Retained placenta
b. Uterine rupture
c. Perineal tear
d. Uterine atony
e. Blood coagulation disorders

A male 65 year old patient came to the Emergency Department with seizures starting from the
right extremities then continue to all four extremities, his eyes and head turned to the right side
and he was non responsive during his seizure, which lasted for 3 minutes. After that, he
regained full consciousness. He had a history of stroke with right hemiparesis since 2 years ago.
EEG revealed abnormality over the left temporal region. What would be the most best drug of
choice treatment for the diagnosis above?
a. Anti stroke drug orally
b. Anti convulsant drug orally
c. Anti status epileptic drug orally
d. Anti seizure drug orally
e. Anti epileptic drug orally

A 42-year-old man was send by policeman to emergency department. When arrived at the
hospital he was anxious, there is massive pool of blood around his right femur. His airway and
breathing were clear, his vital signs: BP 80/60 mmHg, HR 120 beat/min, RR 28 breath/min.
What stages of hemorrhagic shock in this patient?
a. Class IV
b. Class III
c. Class I
d. Class V
e. Class II

A 35-year-old woman come to emergency room with complains of chest pain since 3 hour ago.
She said that the pain is not radiating, sharp on her left chest and felt worse every time she
inhaled. The pain does not alleviated by resting, nor aggravated by exercise. She also had
complains of productive cough over 1 week. She had routinely taking oral contraception. She is
a business woman who just travels from New York to Singapore in a long range flight.
Examination reveals BP 130/90 mmHg, HR 120 x/min, RR 32 x/min, T 38oC. What is the
definitive treatment?
a. Amlodipine
b. Aspirin
c. Oxygen 100% via Nasal prongs 2 L/min
d. Propranolol
51
e. Low-molecular weight heparin
An 18-year-old adolescent girl with a history of depression and opioid dependence presented to
the emergency department after her father found her lying on the floor unresponsive with
shallow respirations. On admission to the emergency department, the patient had a blood
pressure of 95/70 mmHg, heart rate of 50, oxygen saturation was 84% on 100% nonrebreather
mask, and respiration rate was 8 breaths/min. Physical examination showed pinpoint pupils,
weight: 66 kg. Patient is known to have a routine prescription of hydrocodone since she had a
car accident last year. You consider this patient as having an opioid overdose and aim to
administer initial dose of naloxone per intravena. How much naloxone should be administered
to this patient?
a. 2.4 mg
b. 6.6 mg
c. 8.0 mg
d. 4.8 mg
e. 5.2 mg

You are working on your night shift and a 50-year-old woman came to the ER. After the primary
survey was done GCS=15 RR=16 regular HR=75 regular, BP= 130/90, you notice a “dinner fork
deformity” on her left wrist. After further history taking, the patient was fallen onto an
outstretched hand and complained of pain since. what would be your next step?
a. X-Ray then apply inline traction and immobilization device
b. Apply elastic bandage to the left wrist
c. Apply inline traction and immobilization device then continue with x ray
d. Consult with your orthopaedic surgeon
e. Resuscitation fluid 20 cc/kg

A 19-year-old woman presents with feses came out from her vagina. She has just delivered a
3800 g infant vaginally after a 15 hour labor assisted by traditional birth attendance/dukun 1
week ago. The delivery itself was spontaneous after 5 minutes of pushing with no proper
maneuver, the patient said that there is tear at her vagina but the traditional birth attendance
didn’t do anything for the tear. What is the most likely cause of her complaints?
a. Perineal tear 1st degree
b. Perineal tear 3rd degree
c. Perineal tear 4th degree
d. Perineal tear 2nd degree
e. Perineal tear 5th degree

50-year-old woman with a history of high blood pressure comes to the emergency department
because of a severe, throbbing right-sided headache associated with nausea, vomiting and
photophobia. The headache began suddenly without any history of trauma, as a dull ache and
gradually increased in severity over several hours. One hour earlier, suddenly her left side of
the body became paralyzed which remains present. Her temperature is 37.2 C and there is no
resistance on passive flexion of the head. Which of the following is the most likely diagnosis?
52
a. Meningitis
b. Migraine
c. Temporal arteritis
d. Brain abscess
e. Intracerebral hemorrhage

A 56 year-old male with longstanding uncontrolled diabetes was admitted to emergency unit
with decreased consciousness since 2 days ago. He also had wound on the sole of his right foot
since 2 weeks ago. On physical examination he was somnolen, with BP 80 mmHg/palpation,
pulse 120 bpm, respiratory rate 34 times/minute, temperature 38°C. His extremities were cold
and clammy. There was an ulcer in the sole of his right foot, diameter of 5 cm, dirty with pus
and necrotic tissue. The examination to determine which definitive antibiotics to be used is?
a. Procalcitonin
b. Blood leucocyte with differential count
c. Blood culture
d. Wound culture
e. C-reactive protein

A 56 year-old male with longstanding uncontrolled diabetes was admitted to emergency unit
with decreased consciousness since 2 days ago. He also had wound on the sole of his right foot
since 2 weeks ago. On physical examination he was somnolen, with BP 80 mmHg/palpation,
pulse 120 bpm, respiratory rate 34 times/minute, temperature 38°C. His extremities were cold
and clammy. There was an ulcer in the sole of his right foot, diameter of 5 cm, dirty with pus
and necrotic tissue. What primary management for the latest condition?
a. Intubation of endotracheal tube and put on ventilator with control mode
b. Coloid loading of 500 ml
c. Norepinephrin 0.01 mg/kg BW/ minute
d. Crystaloid loading 30 ml/kg BW
e. Send to surgeon for exploratory laparotomy

A young man is brought to the ER involved in the roadway accident and sustaining brachial
artery injury in the ER. Ater ABCD is assessed and secured in the ER, what is the best way to
control the bleeding?
a. Find and ligate the brachial arteryCongestive heart failure
b. Torniquet
c. Direct Pressure
d. 7-0 prolene repair
e. Surgical clamp

A man, 28 years-old, during hospitalization after severe head injury, had the repetition of his
left arm jerking. According to his mother, it was his first attack and he has no history of other
illness. What is your working diagnosis of this attack?
a. Epilepsy
53
b. Acute symptomatic seizures
c. Pseudoseizure
d. Status convulsivus
e. Movement disorder

A 62 –year-old man come to Emergency Department with chief complaint a sudden severe
chest pain 4 hours ago. The substernal chest pain start suddenly while he was watching his
favourite football match on TV. The pain lasted for about 30 minutes, radiated to his jaw and
left arm while he sweated a lot. His wife force him to go seek medical attention although now
he does not feel the pain anymore. He has a history of abnormal fasting glucose test but not
take any medicine regularly. You perform an ECG exam, and order cardiac enzyme. The ECG
report shows a sinus rhythm ECG and no ST segment deviation. What would you do now?
a. Observe the patient while waiting for cardiac enzyme
b. Perform CT scan to find another diagnosis
c. Allow the patient to go home and ask him to come for check up tomorrow
d. Consult Cardiologist for primary percutaneous coronary intervention
e. Consult Cardiologist for emergency echocardiography

Mr. Ahmad, 65-year-old, had an anterior myocardial infarction one year ago. He has shortness
of breath and easily fatigue when walking long distance or climbing more than 2 stairs. With
regular daily activity, he has no marked limitation. What is his functional class?
a. Cannot be determined
b. NYHA class 2
c. NYHA class 3
d. NYHA class 1
e. NYHA class 4

A 68 years old male brought to emergency department due to recurrent brief episode of
unconsciousness in the past 6 months. He had history of hypertension and diabetes mellitus.
The doctors then perform tilt testing, and the result showed blood pressure drop and
bradycardia. What is your diagnosis in this patient?
a. Carotid sinus syncope
b. Orthostatic hypotension syncope
c. Situational syncope
d. Vasovagal syncope
e. Vascular steal syndrome

A 36-year-old male was found unconscious after party with his friends. His friends witnessed
that he consumed heroin and also drink a lot of alcohol. The patient looked bluish and did not
response to any stimulation, he was quickly brought to the Emergency Department. As the
doctor on duty, you suggest the patient had opioid overdose. He has respiratory rate <12
breaths per minute. What are the triad of symptoms of opioid toxicity?
a. Respiratory repression, Pupillary constriction, Hypotension
54
b. Respiratory regression, Pupillary dilatation, Unconsciousness
c. Respiratory restriction, pupillary dilatation, hypertension
d. Respiratory regression, Pupillary dilatation, Hypotension
e. Respiratory depression, Pupillary constriction, Unconsciousness

A male patient, 32 years old had a motorcycle accident and was brought to your emergency
department. Primary survey has been cleared, the patient was in shock with a heart rate of
134x/min and blood pressure of 80/60. Fluid challenge had already been given (1 L of warm
isotonic solution) and the went up to 100/80 for 15 minutes before falling back to 70/50. What
is the most likely cause?
a. Spinal shock
b. Occult bleeding
c. Cardiogenic shock
d. Septic shock
e. Cervical trauma

A 70 y.o. female is admitted to ER at 11 PM with chief complain diffuse/whole severe


abdominal. At first the pain was only in her epigastric area for 4 days and then it spread to
whole abdomen for the last 2 days. She also have continuous progressive fever since this
morning, with very minimal oral intake since this morning also. Since 3 days ago she has
melena, without diarrhea, with normal stool volume and frequency. She also has decreased
micturition. No history of abdominal trauma, no history of previous episode like this. She has
history of chronic gastritis and chronic knee arthritis. She routinely consumed painkiller for her
knee arthritis, bought by the patient herself (self-medication) for more than 10 years. She now
looks sleepy and lethargic with GCS E3M6V5 BP 100/60 mmHg, HR 120x/m, RR 20x/m, Temp
38C. Skin is dry with decreased turgor. Extremity is cool, clammy with normal capillary refill.
Abdomen is distended, negative liver dullness, decreased bowel sound, muscular rigidity is
present with tenderness and rebound tenderness at palpation. Digital Rectal Examination/DRE
shows that she has decreased sphincter tone, smooth mucosa, rectal vault is normal with
tenderness all direction. The punctum maximum of pain is at the epigastric region. At your
gloves after DRE you found black bloody stool with foully smelling. Now for source control and
the definitive treatment, what will you do?
a. Emergency hemodialysis
b. Broad spectrum antibiotics for infection control
c. Consult the surgeon for emergency exploratory laparotomy
d. Emergency upper GI endoscopy for bleeding control
e. Emergency blood and bleeding factor transfusion

A 40-year-old woman presented with subacute onset of quadriparesis of 3 months' duration,


associated with electric shock like pains radiating along the outer aspect of the right arm. On
examination, she had spastic grade 4/5 quadriparesis, the right biceps jerk was absent, and the
other tendon jerks were brisk with extensor planter responses. What is the most likely
diagnosis?
55
a. Subacute combined degeneration of the cord
b. Motor neuron disease
c. Nerve root compression at C3 level
d. Guillain-Barre syndrome
e. Cervical cord compression at C5 level

A girl just quarrel with her boyfriend because She unintentional looked message in an intimate
word and read repeatedly. It’s sender from the other girl. After incident, she try suicide as a cry
for help, to get attention, unusual but not fatal. What’s kind of suicide this girl act?
a. Suicide serious
b. Suicide equivalent
c. Suicide attempt
d. Suicide gamble
e. Suicide gesture

A 30 years old male come to ER at 10 AM with chief complaint of 2 nd degree burn injury at all of
his left hand, left forearm (all circumference) and left elbow joint (all circumference) due to
scald burn injury/ hot water accident, happened 2 hours ago. The pic is shown below:

How many percent is the burn injury?


a. 18.5%
b. 25%
c. 6%
d. 20%
e. 30%

A 78 years old male is admitted to ER with chief complaint severe whole abdominal pain,
starting since 3 days ago. At first he complained of colicky severe abdominal pain, accompanied
with nausea and vomiting, and cannot passage gas, also distended abdomen since 6 days ago.
He denies any fever, but his micturition is decreased since 3 days ago. Previously he had
alteration of his bowel habit since 6 months ago, the stool caliber is decreased and
accompanied with fresh blood, mixed in his feces. No history of chronic diarrhea. Decreased of
weight also noticed by the patient and the family but they don’t know about the number. His
physical exam: He is lethargic, anemic, look cachexic. BP 110/60 mmHg, HR 100x/m, weak but
regular. RR 28x/m, Temp 36.8 C. Body weight 60 kg, Height 170 cm. urine output 30 cc/hour.
His abdomen: distended, no bowel sound, percussion: pain at whole abdomen, palpation:

56
tenderness and rebound tenderness at whole abdomen, muscular rigidity at whole abdomen.
What has happened to this patient?
a. It is an acute abdomen with shock
b. It is a paralytic ileus because the patient has no bowel sound
c. It is an obstructed bowel case that lead to strangulated ileus and peritonitis in the end
d. It is a peritonitis because the patient has distended abdomen
e. It is an obstructed bowel because he has no passage of the stool for 6 days

A 2-years-old boy was found unconsciousness in his house. He accidentally consumed morphine
sustained release tablets, which is usually taken by his grandmother who has chronic pain from
osteoarthritis. The patient was rushed into the triage of Emergency Department. The patient
weighed 15 kg, what is the Naloxone initial dosing in this patient?
a. 1.5 mg
b. 0.6 mg
c. 15 mg
d. 7.5 mg
e. 0.04 mg

A 65 year-old man presents to the emergency department with spinning sensation, vomit,
slurred speech and walking difficulty (ataxia) since 2 days before admission. These symptoms
occur when he was wake up in the morning. He has uncontrolled hypertension. Blood pressure
is 190/110 mmHg and heart rate is 100/min, regular. He has no weakness on his extremities,
but poor balance and coordination on the left side. Which one of the following is the most likely
a cause of his condition?
a. Left Cerebellar artery occlusion
b. Right Internal carotid artery stenosis
c. Cerebral sinus thrombosis
d. Right Medial cerebral artery occlusion
e. Rupture aneurysm of left anterior cerebral artery

A 37 year old man is having habit of drinking of beer. After some months, his habit becomes
heavier and he drinks wine, Whiskey and Vodka. Few months later, he was admitted to a
hospital because of an accident where he banged his head when crossing a wooden bar where
he fell unconscious for few minutes. He was advised to be hospitalized. On the third day, his
hand began to shake and the whole body also trembles After being put on some drugs his
shaking subsided and he was discharged after being pronounced as recovered from his
ailments. What was the condition causing his shaking?
a. Blackouts
b. Post Traumatic Stress Disorder
c. Delirium tremens
d. Acute Stress Disorder
e. Dementia

57
A 32-year-old woman presents with a grade 2 open midshaft femoral shaft fracture as the result
of a high-speed motor vehicle collision. Concomitant injuries include a high-grade splenic
laceration requiring splenectomy as well as a subdural hematoma that requires monitoring and
maintenance of cerebral perfusion pressure. Which of the following is the earliest appropriate
management of the femoral shaft fracture at this time?
a. Performed immediate surgical amputation to minimized blood loss
b. Immediate ORIF with plate and screw
c. Irrigation and debridement of the open fracture continued with external fixation
d. Not started any treatment for the fracture until spleen laceration and subdural hematoma
are healed
e. Primary wound closure and immobilization

A 6-year-old boy developed seizure and fever to 39⁰C. He had multiple generalized seizures,
ranging from 10 to 15 minutes. The last seizure had occured for more than 20 minutes and had
not stopped. His parents had given 2 doses of rectal diazepam at home. The boy was brought to
emergency department. An intravenous diazepam was given but the seizure persisted. What is
the most appropriate management for this patient?
a. Give 20 mg/kg intravenous phenytoin
b. Do a lumbar puncture
c. Do an electroencephalography
d. Give another dose of intravenous diazepam
e. Give 5 mg/kg intravenous phenobarbital

I feel like I'd be struck by lightning if I didn't turn three times to the right. Or to believe my sister
was going to have an accident because I didn't stop at the odd-numbered stairs. My feet, my
hands, always don't stop holding things over and over again until they feel right. Often in my
heart I count odd numbers until my feelings become calmer. What disorder is this?
a. Avoidant Personality Disorder
b. Obsessive-Compulsive Disorder
c. Obsessive-Compulsive Personality Disorder
d. Narcissistic Personality Disorder
e. Schizotypal Personality Disorder

Mr Hendry, 34 y.o. with history of head trauma in motor vehicle accident 2 hours before, was
brought to ER with unconsciousness. Now he is alert with GCS 15 but he has a very bad
headache. his skull X ray shows linier fracture at the left temporal region, and the physician
decided to do observation for 6 hours, but after 2 hours he became unconscious again. His left
arm and leg became inactive compare to the right side. How do you explain this?
a. The right midbrain is compressed either by blood clot or the swelling brain
b. The left midbrain is compressed either by blood clot or the swelling brain
c. The left brain are pushed and shift to the right and the right midbrain are compressed the
tentorial edge

58
d. Right brain are pushed and shift to the left and the left midbrain are compressed the
tentorial edge
e. The hematoma cause compression on both side of the brainstem

A 30 year old man injured his thoracic spine in motor vehicle accident 2 years ago. Now he still
has pain and thermal sensation loss on part of his left body and propioception loss in his right
foot. There is still a paralysis of the right lower extremity as well. This patient most likely has
which of the following spinal cord conditions?
a. Hemisection syndrome
b. Anterior cord syndrome
c. Posterior collum syndrome
d. Central cord syndrome
e. Complete transection

A 62 –year-old man come to Emergency Department with chief complaint a sudden severe
chest pain 4 hours ago. The substernal chest pain start suddenly while he was watching his
favourite football match on TV. The pain lasted for about 30 minutes, radiated to his jaw and
left arm while he sweated a lot. His wife force him to go seek medical attention although now
he does not feel the pain anymore. He has a history of abnormal fasting glucose test but not
take any medicine regularly. What is the most appropriate diagnosis now?
a. Acute pericarditis
b. Acute coronary syndrome
c. Aortic regurgitation
d. Mitral Stenosis
e. Psychological chest pain

A 78 years old male is admitted to ER with chief complaint severe whole abdominal pain,
starting since 3 days ago. At first he complained of colicky severe abdominal pain, accompanied
with nausea and vomiting, and cannot passage gas, also distended abdomen since 6 days ago.
He denies any fever, but his micturition is decreased since 3 days ago. Previously he had
alteration of his bowel habit since 6 months ago, the stool caliber is decreased and
accompanied with fresh blood, mixed in his feces. No history of chronic diarrhea. Decreased of
weight also noticed by the patient and the family but they don’t know about the number. His
physical exam: He is lethargic, anemic, look cachexic. BP 110/60 mmHg, HR 100x/m, weak but
regular. RR 28x/m, Temp 36.8 C. Body weight 60 kg, Height 170 cm. urine output 30 cc/hour.
His abdomen: distended, no bowel sound, percussion: pain at whole abdomen, palpation:
tenderness and rebound tenderness at whole abdomen, muscular rigidity at whole abdomen.
For the first management, what will you do as the ER doctor?
a. Give the patient intravenous albumin
b. Give the patient Packed Red Cell Transfusion
c. Send the patient to ICU since he has sepsis condition
d. NPO and fluid rehydration
e. Send the patient for emergency laparotomy
59
A young adult, 16 year-old is watching tv-program, suddenly stops her activity, unresponsive,
then falls down and starts convulsion. She had previous same attacks, twice in a month since
two months before. When there is no attack, she is normal. What is the working diagnosed of
this case?
a. Behavior disorder
b. Acute symptomatic seizure
c. Epilepsy
d. Status convulsivus
e. Status epilepticus

A 55 year old female was picked up at home by Emergency doctor because of unconsiousness.
According to the doctor, when he arrived, he found that SpO2 of the patient was 77%. RR
23x/min, HR 115x/min, BP 100/60. Oxygen was given maximally and the SpO2 became
increased to 85- 90%. blood Gas Analysis was performed and showed pH 7,31, pCO2 65, pO2
50, HCO3 30, BE +5, SpO2 90%, with NRM 15 lpm. Chest Xray showed bilateral opacities on both
side of the lung. What is the best explanation of the initial condition?
a. Retained intravascular protein.
b. Recruitment of neutrophils
c. Alveolar Capillary injury
d. Tight junction between alveolar and epthelial cells.
e. Diffuse Alveolar Damage

A 42-year-old man was send by policeman to emergency department. When arrived at the
hospital he was anxious, there is massive pool of blood around his right femur. His airway and
breathing were clear, his vital signs: BP 80/60 mmHg, HR 120 beat/min, RR 28 breath/min. You
also observe that right femur is edematous, with significant deformity is seen. He still has good
distal capillary refill and good pulsation of right dorsalis pedis artery. After few hours, the
patient still alert with HR 100x/m, complained of severe pain and the femoral region looks more
swollen. You observe that there is decreased pulse in the right popliteal artery, the skin of the
right femur is pale and cold. What is the possible cause?
a. The patient is having open fracture grade IIIc at the left femur
b. The patient was in haemorrhagic shock, so it is possible that he is in transient response of the
haemorrhagic shock
c. The patient is having femoral fracture, and the segment become more displaced because of
the patient’s movement and now the segment of fracture contused the femoral artery, causing
injury to the femoral artery
d. He is having acute compartment syndrome in the right femoral region which compressed the
right popliteal artery
e. He is having septic shock due to the open wound at the right thigh

A 23-year-old man is brought immediately to the emergency department from the hospital' s
parking lot where he was shot in the lower abdomen. Examination reveals a single bullet
60
wound. He is breathing shakily and has a thready pulse. However, he is unconscious and has no
detectable blood pressure. Optimal immediate management is to
a. Perform emergency Abdominal and pelvic CT Scan
b. Perform intubation
c. Initiate transfusion of Fresh Frozen Plasma and Whole blood cell
d. Initiate primary survey and transfer the patient to the operating room
e. Give him oxygen and insert urinary catheter

What should a three years-old boy with three episodes of febrile seizure within the past year
best be treated with?
a. Diazepam
b. No medication
c. Carbamazepine
d. Phenobarbital
e. Valproic acid

A 55 year old female was brought to Emergencys because of unconsiousness and SpO2 was
77%. Oxygen was given maximally and the SpO2 became increased to 90%. blood Gas Analysis
was performed and showed pH 7,31, pCO2 65, pO2 50, HCO3 30, BE +5, SpO2 90%, with NRM
15 lpm. What is the best action to this patient?
a. Non invasive ventilation
b. Non Rebreathing mask
c. High flow high ventilation mask
d. Intubation
e. High flow nasal canul

A 29 year old woman presents with 3 days of dark spotting and mild cramping at 9 weeks
gestation. An ultrasound notes a viable 9 weeks gestation with no obvious problems in the
uterus or ovaries. On obstetrical examination, the cervix is closed, smooth with minimal
bleeding through the ostium. What is the most likely diagnosis?
a. Premature labor
b. Inevitable abortion
c. Incomplete abortion
d. Imminen abortion
e. Threatened abortion

A 30-year-old woman is brought into your clinic with burns affecting both of her entire upper
extremities and the anterior surfaces of both legs after spilling boiling soup from the stove
while cooking approximately two hour ago. Her weight is 48 kg, height 150 cm, blood pressure
90/60 mmHg, heart rate 100 bpm, respiratory rate 20 x/min. Using Wallace's Rule of Nines
what is the percentage area burnt?
a. 45%
b. 18%
61
c. 27%
d. 54%
e. 36%

An electrician is electrocuted by a downed power line after a thunderstorm. He apparently


made contact with the wire at the level of the right mid thigh. In the emergency department,
his vital signs are normal and no dysrhythmia is noted on ECG. On examination, there is an exit
wound on the bottom of the right foot. His urine is positive for blood by dip stick but no RBCs
are seen microscopically. Initial management should include:
a. Admission to the intensive care unit for observation
b. Aggressive fluid infusion
c. Intravenous pyelography
d. Immediate angiography
e. Debridement of necrotic muscle

A 37 year old G5P3A1 came to the emergency department because of vaginal bleeding, and
abdominal cramps. History is unrevealing except for an induced abortion. Physical examination reveals
a BP of 110/ 70 mmHg, pulse 120 x/mnt, and temperature 38.50C. The abdomen is tender with slight
rebound in the lower quadrants. The pelvic examination reveals blood in the vault and a foul smelling
discharge from the cervix, which is dilated to 1 cm. The uterus is 8 weeks size and tender. What do you
think happen in this patient and what is the treatment?
a. Molar pregnancy and laparotomy
b. Ectopic pregnancy and laparotomy
c. Septic abortion and curettage
d. Uterine perforation and laparotomy
e. Missed abortion and curettage

A 45 year old woman was found unconsciousness after falling from a bicycle due to hitting a tree. At
Emergency Department, her BP 110/70, RR 10 times/min, HR 100x/m, SpO2 90. CT scan showed fracture
on cervical spine. As an emergency doctor, what should you anticipate to this patient?
a. Ventilation impairment
b. Diffusion impairment
c. Restrictive lung
d. Perfusion impairment
e. Obstructive lung

Recently, We heard news from television that there was a bomb exploded at a big mall in the city. As the
consequence, many people was died there at that moment overall. Which of the answer is the most
correct one in this case?
a. Attempt suicide
b. Suicide attack
c. Mass suicide
d. Murder suicide
e. Assisted suicide

62
18 y.o. male is brought to the ER after involved in roadway accident. His HR is 150x/m with BP
70/palpation, After initial fluid resuscitation with 2 liters of lactated Ringer, on Focussed Abdominal USG
for Trauma (FAST) exam demonstrates very large collection of blood in the abdomen. his right leg is also
severely mangled. Which is the most appropriate next step?
a. CT of head, neck and torso, and lower extremity
b. Arteriography and Intravenous antibiotic
c. Surgical consult to explore leg fracture
d. ICU for further volume resuscitation and stabilization
e. emergency Exploratory Laparotomy to find and control abdominal bleeding

Patient came to emergency room due to profuse vaginal bleeding since yesterday. She claimed that she
is pregnant for 3 months. She has just passed some tissues prior to consult and after that the bleeding
diminished. She is on stable vital signs. On physical examination, the abdomen was soft and non-tender.
The cervix was soft and closed, with minimal bleeding from vagina. Ultrasound has been done and the
result is an empty uterus and both adnexas within normal limits. What do you think happen in this
patient and what is the treatment?
a. Complete abortion and observation
b. Incomplete abortion and observation
c. Complete abortion and curettage
d. Incomplete abortion and curettage
e. Missed abortion and observation

A girl just quarrel with her boyfriend because She unintentional looked message in an intimate word and
read repeatedly. It’s sender from the other girl. After incident, she try suicide as a cry for help, to get
attention, unusual but not fatal. What therapy gave properly?
a. Pharmacotherapy
b. Hypnotherapy
c. Group therapy
d. Social therapy
e. Psychotherapy

A 67 year-old female with longstanding uncontrolled diabetes was admitted to emergency unit with
decreased consciousness since 2 days ago. She also experienced productive cough since 2 weeks ago. On
physical examination he was somnolen, with BP 80 mmHg/palpation, pulse 120 bpm, respiratory rate 34
times/minute, temperature 38°C. Her extremities were cold and clammy. There was rales on thoracic
examination, her peripheral oxygen saturation was 90%. The primary cause of her vasculatory shock
was?
a. Movement of intravascular fluid due to increased permeability caused by generalized inflammation
b. Unability of the heart to pump blood due to longstanding diabetes and coronary arterial disease
c. Decreased intake due to decreased consciousness
d. General vasodilatation caused by vasoplegia
e. Unability of the heart to pump blood caused by ischemia to the heart muscle due to low oxygen
saturation
A 36-year-old woman is admitted to company clinic following a workplace accident. She says that her leg
and foot is splashed by a liquid from a falling chemical container. She is in great pain and crying for your
help. What is the first action you take?
a. Give pain reliever
63
b. Test the liquid on affected area with litmus paper
c. Perform a primary survey of the patient
d. Contact the plastic surgeon on call urgently
e. Commence water irrigation

A teenage bicycle rider is hit by a truck traveling at a high rate of speed. In the emergency department,
she is actively bleeding from open fractures of her legs, and has abrasions on her chest and abdominal
wall. Her blood pressure is 80/50 mm Hg, heart rate is 140 beats per minute, respiratory rate is 8
breaths per minute, and GCS score is 6. The first step in managing this patient is to
a. Perform endotracheal intubation and ventilation
b. Apply the PASG and inflate the leg compartments
c. Insert a central venous pressure line
d. Administer 2 liters of crystalloid solution
e. Obtain a lateral cervical spine x-ray

A 22 yo male sustained compartment syndrome at his left calf following a closed tibial fracture. Which
compartment is most commonly affected by compartment syndrome?
a. Superficial posterior compartment
b. Deep posterior compartment
c. Anterior compartment
d. Lateral compartment
e. Peroneal compartment

A five years-old girl developed a high fever, of 40C, and increasing difficulty breathing within a 10-hour
period. She refused to drink or eat, and she cried only minimally despite her ill-looking appearance. Her
vital status: disoriented, HR 140x/m, RR 48x/m, temp 40C, O2 sat 80%. There is some drooling of saliva
down her chin, and a soft inspiratory stridor is heard. She had no preceding coryzal symptoms. What is
the most appropriate next step of management?
a. Order a chest X ray
b. Take a throat swab for culture and sensitivity
c. Arrange urgent intubation
d. Give antibiotics
e. Give nebulized adrenaline to the patient

A 27 year old girl attempted suicide by eating ten pills all at once. Her mother found her diary saying
that she really wants to be dead because her boyfriend has another girl and leaved her. What is the
most probably diagnosis?
a. General Anxiety Disorder
b. Panic Disorder
c. Post Traumatic Stress Disorder
d. Schizophrenic
e. Major Depressive Disorder

A 30-year-old woman experiences impulsive wishes to slash her wrists, generalized anxiety, social
phobia and dissociation. Although examination reveals no psychotic symptoms, she reports having
recurring flashbacks of childhood sexual and physical abuse. What is the most likely diagnosis?
a. Depression
64
b. Generalized anxiety disorder
c. Schizophrenia
d. Obsessive-compulsive disorder
e. Post-traumatic stress disorder

A 30-year-old man is struck by a car traveling at 80km/hour.He has obvious fractures of the left tibia
near the knee, pain in the pelvic area, and severe dyspnea. His heart rate is 180 beats per minute, and
his respiratory rate is 48 breaths per minute with no breath sounds heard in the left chest. A tension
pneumothorax is relieved by immediate needle decompression and tube thoracostomy. Subsequently,
his heart rate decreases to 140 beats per minute, his respiratory rate decreases to 36 breaths per
minute, and his blood pressure is 80/50 mmHg. Warmed Ringer's lactate is administered intravenously.
The next priority should be to:
a. Perform external fixation of the pelvis
b. Call family to give emergency blood transfusion for the patient
c. Consult surgeon for emergency operation
d. Obtain abdominal and pelvic CT scans
e. Full blood examination, including Lactate and Blood gas analysis

A 70 y.o. female is admitted to ER at 11 PM with chief complain diffuse/whole severe abdominal. At first
the pain was only in her epigastric area for 4 days and then it spread to whole abdomen for the last 2
days. She also have continuous progressive fever since this morning, with very minimal oral intake since
this morning also. Since 3 days ago she has melena, without diarrhea, with normal stool volume and
frequency. She also has decreased micturition. No history of abdominal trauma, no history of previous
episode like this. She has history of chronic gastritis and chronic knee arthritis. She routinely consumed
painkiller for her knee arthritis, bought by the patient herself (self-medication) for more than 10 years.
She now looks sleepy and lethargic with GCS E3M6V5 BP 100/60 mmHg, HR 120x/m, RR 20x/m, Temp
38C. Skin is dry with decreased turgor. Extremity is cool, clammy with normal capillary refill. Abdomen is
distended, negative liver dullness, decreased bowel sound, muscular rigidity is present with tenderness
and rebound tenderness at palpation. Digital Rectal Examination/DRE shows that she has decreased
sphincter tone, smooth mucosa, rectal vault is normal with tenderness all direction. The punctum
maximum of pain is at the epigastric region. At your gloves after DRE you found black bloody stool with
foully smelling. You need to stabilize the patient, what do you do? (prioritize your action)
a. Give fluid and blood to the patient, antibiotics and analgetics
b. Non per oral, Give oxygen and IV Fluids, abdominal decompression, IV antibiotics and analgetic
c. Intubate and send the patient to the ICU
d. Emergency consult to surgeon for emergency exploratory laparotomy
e. Lab check and abdominal xray, abdominal CT if needed

A 24-year-old boxer was brought to emergency room with unconsciousness. Based on paramedic’s story
he was knocked out during a match after getting hit on his temple. He was initially unconscious but
regains his consciousness several minutes later. On the way to the hospital there was traffic jam, and he
appears to be drowsier until he gives no response. On examination you found under pain stimuli his eyes
are not reacting, he produce incomprehensible sounds, withdrawal reflex from the stimuli and unequal
pupil. What is your first step to help this patient?
a. Obtain emergency head CT-scan
b. Begin CPR sequences
65
c. Refer to a neurosurgeon
d. Secure definitive airway
e. Give him oxygen 10 L/min via non-rebreathing mask

A 72 year-old man presents to the emergency department with right-sided weakness and speech
difficulty (global aphasia) suddenly since 5 hour before admission while he was brushing his teeth. He
has uncontrolled diabetes mellitus type 2, and he is on warfarin for atrial fibrillation. Blood pressure is
170/90 mmHg and heart rate is 66/min, irregular. He has right sided weakness. A bruit is heard over the
left carotid artery. Brain CT scan showed signs of left medial cerebral artery occlusion with mild midline-
shift. Which of the following is the most appropriate as early specific treatment?
a. Intravenous antihypertension
b. Mechanical thrombectomy
c. Loading oral antiplatelet
d. Subcutaneous anticoagulant
e. Intravenous thrombolysis with r-TPA

Mr. Hendry, 34 years old male, got a motor vehicle accident without helmet and his head hit the road.
He become unconscious for 2 hours, and awake when he is in ER. When he came, his blood pressure is
120/80 mmHg, pulse 88 x/minutes, respiratory rate was 12 x/ minutes. He become conscious but with a
very bad headache. He forgot how he fell but he is fully oriented. All of his motoric and sensoric is fine.
His general condition is unremarkable. What would your thought about his cortico spinal tract at that
moment?
a. Cannot be assessed clinically
b. Probably partial injured
c. It’s injured
d. Cannot be concluded yet
e. It’s intact

A 30-year-old woman is brought into your clinic with burns affecting both of her entire upper extremities
and the anterior surfaces of both legs after spilling boiling soup from the stove while cooking
approximately two hour ago. Her weight is 48 kg, height 150 cm, blood pressure 90/60 mmHg, heart
rate 100 bpm, respiratory rate 20 x/min. Using Parkland formula how much fluid you need to administer
EACH hour for the next 6 hour?
a. 576 cc
b. 3456 cc
c. 132 cc
d. 378 cc
e. 824 cc

A-47 year old man with a history of taking 250-300mL wine with high ethanol concentration. He came to
emergency department due to black, bloody stools, and vomit that looks like coffee grounds after
consuming aspirin for around a week. The doctor thought that might be because of local effect
combination of aspirin-alcohol at gastric mucosal. But actually, the problem also occur when such
patients took many other drugs like benzodiazepine (clonazepam, alprazolam). Acute ethanol ingestion
may potentiate the CNS effects of benzodiazepine or tolerance of the drug may develop with chronic
alcohol abuse. Which of the following is the most likely mechanism of alcohol can make that situations?
66
a. Decrease intestinal transit time
b. Desensitizes receptors of other drugs
c. Inhibit renal excretion with maximizing other drugs resorption
d. Binding of other drugs
e. Decrease ability of hepatocyte to metabolize other drugs
A 55 year old male came to Emergency department because of dry cough and continuous fever since 3
days ago. He denied of shortness of breath. He had lunch with his friends 1 week ago and currently
some of them experience the same symptoms. Chest X ray showed bilateral ground glass opacities. On
physical examination, BP 130/90, RR 28x/min, HR 105x/min, temperature 39⁰C. Blood Gas Analysis
showed pH 7.5f2, PCO2 25, pO2 50, HCO3 19, BE -6, SpO2 88%. What is your initial management of this
patient?
a. High Flow Nasal Canule
b. Non Rebreathing Mask
c. Rebreathing Mask
d. Thorax PA
e. Bagging

A female 29 year old patient came to the Emergency Department with generalized tonic clonic seizure
that lasted for 2 minutes. The seizure continued for 3 times with no regain of consciousness between
seizures. What would be your diagnosis now?
a. Status epilepticus
b. Epilepsy focal to bilateral tonic clonic with impaired consiousness
c. Epilepsy focal with impaired awareness
d. Acute symptomatic seizure
e. Epilepsy generalized onset

Mr Hendry, 34 y.o. with history of head trauma in motor vehicle accident 2 hours before, was brought to
ER with unconsciousness. Now he is alert with GCS 15 but he has a very bad headache. his skull X ray
shows linier fracture at the left temporal region, and the physician decided to do observation for 6
hours, but after 2 hours he became unconscious again. Now his BP increase to 150/95 mmHg, pulse up
to 100 x/minutes, respiratory rate up to 20 x/minutes. The pupil on the left 4 mm/ very slow response to
the light, and right pupil 2 mm response to light. His left arm and leg are inactive compare to the right
side. What do you think the most probable cause of his unconsciousness?
a. The brain is swelling and compressing the brainstem
b. The brainstem itself is swelling
c. There is a hematoma that cause compression of left brainstem
d. There is a secondary brain injury due to prolonged hypoxia
e. There is a bleeding in the pons

A 37 year old man came to Emergency & Accident Unit with feeling of apprehension, fear and
palpitation, even mounting to feeling of impending death. After being examined by doctor, the results
showed that nothing was wrong. After3 weeks, he got the same feelings, and he had to go to the
hospital again. But this time the Emergency staffs were busy, and he was kept to wait. Until about 15-20
minutes he had not been attended to, but his fear and palpitation subsided and became calm by it.
What would you think about the suffering that this person had?
a. Phobic anxiety disorder
b. Neurotic disorder
67
c. Somatization disorder
d. Generalized Anxiety Disorder (GAD)
e. Panic disorder

A 65-year-old gentleman presented to Emergency Department in unconscious condition. His ECG is


attached. He has no pulse. Which of the following is the most appropriate intervention?

a. Adenosine triphosphate intravenous


b. Valsava Maneuver
c. Thrombolytic therapy
d. Carotid Message
e. DC shock

A 47 year old male was brought to emergency due to history of brief unconsciousness. The family told
that the patient was suffering from edema in his both leg since a week ago, and he was prescribed
Furosemide for his problem. Physical examination showed normal heart sound and normal vesicular
sound on both lungs. What do you suggest causing the brief unconscious in this patient?
a. Drug induced pro-arrhythmias
b. Pressure to carotid sinus
c. Drug induced orthostatic hypotension
d. Stimulation to trigeminal nerve
e. Hyperactivity of vasovagal reflex

A 56-year-old man with suddenly weakness of his left extremities and dysarthria. He was smoker and
has uncontrolled hypertension in the last 5 years ago. Patient alert and conscious, Blood pressure
150/90 mmHg, HR 80 x/m regular, RR 16x/m, temperature 36,7 C. Brain CT scan showed lacunar infarct
at right internal capsule. Which of the following most likely as a caused of his condition?
a. Thrombosis at branch of right medial cerebral artery
b. Space occupying lesion at left basal ganglia region
c. Ruptures small aneurysm at left medial cerebral artery
d. Cardioembolic stroke due to occlusion of right internal carotid artery
e. Unruptured aneurysm at branch of right medial cerebral artery

Yuli was harshly treated by her husband from the beginning of her marriage, about three years ago.
Quarrel after quarrel they went through at their rented house in Jakarta, often accompanied by harsh
words and often beaten, until finally once Yuli suffered the miscarriage of her first child. Her husband
later apologized and promised to change his habits. The above events are
a. Physical Abuse, Emotional Abuse, Verbal Abuse
68
b. Physical Abuse, Verbal Abuse
c. Economic Abuse, Emotional Abuse, Verbal Abuse
d. Emotional Abuse, Drug Abuse, Verbal Abuse
e. Verbal Abuse Only
A 24 year old woman (gravida 2, para 0, abortus 1) is seen in the emergency department because of
vaginal bleeding, and abdominal cramps. Her last menstrual periode was 10 weeks ago. History is
unclear except for an induced abortion. Physical examination reveals a BP of 110/ 70 mmHg, pulse 12,
and temperature 38.5 C. The abdomen is tender with slight rebound in the lower quadrants. The pelvic
examination reveals blood in the vault and a foul smelling discharge from the cervix, which is dilated to 1
cm. The uterus is 8 weeks size and tender. What is the most likely diagnosis?
a. Pelvic inflammatory disease
b. Incomplete abortion
c. Ectopic pregnancy
d. Induced abortion
e. Septic abortion

Mr Hendry, 34 y.o. with history of head trauma in motor vehicle accident 2 hours before, was brought to
ER with unconsciousness. Now he is alert with GCS 15 but he has a very bad headache. his skull X ray
shows linier fracture at the left temporal region, and the physician decided to do observation for 6
hours, but after 2 hours he became unconscious again. The pupil on the left 4 mm/ very slow response
to the light, and right pupil 2 mm response to light. His left arm and leg are inactive compare to the right
side. What do you need to do to prove your thinking about this phenomenon in the pupil?
a. Do another skull X ray to see the changes
b. Do CT scan of the head
c. Do a lumbal puncture to see whether the is a blood stained CSF
d. Do MRI of the head
e. Do the blood gas analysis

A 56 year-old male with longstanding uncontrolled diabetes was admitted to emergency unit with
decreased consciousness since 2 days ago. He also had wound on the sole of his right foot since 2 weeks
ago. On physical examination he was somnolen, with BP 80 mmHg/palpation, pulse 120 bpm,
respiratory rate 34 times/minute, temperature 38°C. His extremities were cold and clammy. There was
an ulcer in the sole of his right foot, diameter of 5 cm, dirty with pus and necrotic tissue. Management
to control metabolic condition in this patient is?
a. Prevention thrombosis with LMWH or UFH
b. Control of blood glucose with subcutaneous insulin
c. Prevention of hypoxemia with high flow oxygen
d. Control of infection with broad spectrum antibiotic such as carbapenem
e. Control of blood glucose with intravenous insulin

A female 29 year old patient came to the Emergency Department with generalized tonic clonic seizure
that lasted for 2 minutes. What will be your first drug of choice treatment for this patient?
a. Phenobarbital intravenously
b. Carbamazepine intravenously
c. Phenytoin intravenously
d. Diazepam intravenously
e. Propofol intravenously
69
A 37 year old man falls 4 meters off of a roof and fractures his cervical spine, causing damage at the C4
level. He is initially a flaccid quadriplegic with areflexia. This areflexia and flaccidity evolve into
hyperreflexia and spasticity within 2 weeks later. What is the cause of this phenomenon?
a. Spinal cord compression
b. Spinal shock
c. Spinal cord hypoxic
d. Spinal cord laceration
e. Spinal cord contusion

A 28 year-old man is brought to the Emergency Department by ambulance following a generalized


seizure (which has been witnessed by a competent healthcare professional). He does not have epilepsy
and has never had a seizure before. The patient has now recovered and his GCS is 15. What feature
would indicate that the patient should have an urgent CT head scan?
a. Aspirin therapy
b. Documented hypoglycaemia
c. Alcoholism
d. Family history of epilepsy
e. Focal neurological deficit

A 24-year-old boxer was brought to emergency room with unconsciousness. Based on paramedic’s story
he was knocked out during a match after getting hit on his temple. He was initially unconscious but
regains his consciousness several minutes later. On the way to the hospital there was traffic jam, and he
appears to be drowsier until he gives no response. On examination you found under pain stimuli his eyes
are not reacting, he produce incomprehensible sounds, withdrawal reflex from the stimuli and unequal
pupil. What is the most likely diagnosis of this patient?
a. Subarachnoid hemorrhage
b. Epidural hemorrhage
c. Intracerebral hemorrhage
d. Brain contusion
e. Subdural hemorrhage

A 45 year old woman was found unconsciousness after falling from a bicycle due to hitting a tree. At
Emergency Department, her BP 120/80, RR 10 times/min, HR 88x/m, SpO2 90. CT scan showed fracture
on cervical spine. What is the best explanation of this condition?
a. Obstruction of large airway
b. Leakage of alveoli
c. Destruction of small airway
d. Peripheral resistance increase
e. Lung compliance decrease

A 29-year-old woman presents with massive post partum bleeding. She is gravida 3 para 3 and has just
delivered a 4200 g (9 lb 4 oz) infant vaginally after a 5 hour labor. The delivery itself was spontaneous
after 15 minutes of pushing. The placenta was delivered 5 minutes later with minimal traction. What is
the most likely cause of her bleeding?
a. Coagulation defect
b. Vaginal lacerations
70
c. Retained products of conception
d. Uterine atony
e. Chorioamnionitis
A 34 year old female came to your Emergency department with profuse bleeding. She has just deliver a
twin baby half an hour ago weigh 2800 grams and 2600 grams, this is her 6 th pregnancy. The patient is
unconscious, BP 80/palp, HR 110 beats /min, when you inspect the vagina, heavy bleeding still occurs.
You decided to administered misoprostol to stop the profuse bleeding. What is the maximum dose of
misoprostol that you can admit in 24 hours?
a. 1200 µg
b. 800 µg
c. 600 mg
d. 1200 mg
e. 800 mg

A 36-year-old male was found unconscious after party with his friends. His friends witnessed that he
consumed heroin and also drink a lot of alcohol. The patient looked bluish and did not response to any
stimulation, he was quickly brought to the Emergency Department. As the doctor on duty, you suggest
the patient had opioid overdose. He has respiratory rate <12 breaths per minute. What mechanism
cause the respiratory rate <12 breaths per minute in this case?
a. Irreversible brain damage causing respiratory proble
b. Infection from the drug use causing severe sepsis
c. Alcoholism causing liver failure
d. Previous lung problem in the patient
e. Respiratory depression caused by opioid

A 55 year old female was picked up at home by Emergency doctor because of unconsiousness.
Respiratory rate 12x/minutes, shallow, HR 120x/min, BP 110/70, SpO2 77 %, room air. Both of the
extremities were cold. As a emergency doctor, what is the initial management of this patient?
a. Nebulizer
b. High Flow Nasal Canul
c. Bagging
d. Continuous Positive Airway Pressure
e. Put IV line

A 70 y.o. female is admitted to ER at 11 PM with chief complain diffuse/whole severe abdominal. At first
the pain was only in her epigastric area for 4 days and then it spread to whole abdomen for the last 2
days. She also have continuous progressive fever since this morning, with very minimal oral intake since
this morning also. Since 3 days ago she has melena, without diarrhea, with normal stool volume and
frequency. She also has decreased micturition. No history of abdominal trauma, no history of previous
episode like this. She has history of chronic gastritis and chronic knee arthritis. She routinely consumed
painkiller for her knee arthritis, bought by the patient herself (self-medication) for more than 10 years.
She now looks sleepy and lethargic with GCS E3M6V5 BP 100/60 mmHg, HR 120x/m, RR 20x/m, Temp
38C. Skin is dry with decreased turgor. Extremity is cool, clammy with normal capillary refill. Abdomen is
distended, negative liver dullness, decreased bowel sound, muscular rigidity is present with tenderness
and rebound tenderness at palpation. Digital Rectal Examination/DRE shows that she has decreased
sphincter tone, smooth mucosa, rectal vault is normal with tenderness all direction. The punctum

71
maximum of pain is at the epigastric region. At your gloves after DRE you found black bloody stool with
foully smelling. What do you think happened with the patient?
a. Diffuse peritonitis due to perforated colonic tumor
b. Diffuse peritonitis due to perforated peptic ulcer, now with sepsis, possibly severe dehydration
c. Acute bowel obstruction due to colonic tumor
d. Acute appendicitis with perforation, causing diffuse peritonitis
e. Hypovolemic shock due to bleeding from the gastric ulcer

A 78 years old male is admitted to ER with chief complaint severe whole abdominal pain, starting since 3
days ago. At first he complained of colicky severe abdominal pain, accompanied with nausea and
vomiting, and cannot passage gas, also distended abdomen since 6 days ago. He denies any fever, but
his micturition is decreased since 3 days ago. Previously he had alteration of his bowel habit since 6
months ago, the stool caliber is decreased and accompanied with fresh blood, mixed in his feces. No
history of chronic diarrhea. Decreased of weight also noticed by the patient and the family but they
don’t know about the number. His physical exam: He is lethargic, anemic, look cachexic. BP 110/60
mmHg, HR 100x/m, weak but regular. RR 28x/m, Temp 36.8 C. Body weight 60 kg, Height 170 cm. urine
output 30 cc/hour. His abdomen: distended, no bowel sound, percussion: pain at whole abdomen,
palpation: tenderness and rebound tenderness at whole abdomen, muscular rigidity at whole abdomen.
What is likely to be the cause of the case above?
a. Obstructed sigmoid due to volvulus
b. Perforated left tumor colon
c. Perforated left colon due to invagination
d. Perforated splenic abscess
e. Perforated gastric ulcer to Helicobacter pylori infection

A 26-year-old woman present to Emergency Department after giving birth to a healthy 2800 gr baby
with help from Traditional birth attendance/dukun yesterday. Her family says that after the baby was
delivered, it took nearly 1 hour until the placenta is delivered. She is drowsy and less responsive, she is
hypotensive, BP 60/40 mmHg, HR 136, RR 28 x/min, with cool and clammy skin. She is brought because
there is continuous trickle of blood from her vaginal. Which of the following is the emergency step in
managing this patient?
a. Fluid resuscitation
b. Vitamin K injection
c. Antibiotics
d. Elective evacuation of the uterus
e. Analgesia

A 53 years old patient complained chest pain admitted to Emergency room. After performed a series of
physical examination, an ECG test revealed an anterior myocardial infarction. What artery is the most
responsible for that condition?
a. Marginal Coronary Artery
b. Circumflexa artery
c. Posterior Descending interventricular artery
d. Right Coronary Artery
e. Left anterior descending artery

72
A 19 y.o. girl went to ER at 1.00 AM due to severe right abdominal pain since 1 day ago. It started in the
right abdominal region, spread to left abdominal region and to whole abdominal region. She complained
of nausea and loss of appetite, but no vomiting. She denied fever, changes on defecation nor
micturition. She is on her third day period, but according to her it’s not normal since the amount is less
(spotting). Usually she has regular and normal period, no history of severe pain like this. What is
first properquestion you need to ask her?
a. Are you sexually active ?
b. Any history of previous surgery?
c. Are you married?
d. Are you pregnant?
e. Any history of gynecologic problems?

73

You might also like